University of Texas 2000 Answers

 

 

 

 

 

 

 

 

 

 

 


1.


Best answer: B


A: Resnick makes no mention of cystic lucencies from triangular fibrocartilage tears. Wheeless' Textbook of Orthopaedics (online) describes Radiographic findings of Triangular fibrocartilage tear
- Triple Injection Arthrography:
- study of choice, but can often miss peripheal meniscus tears;
- identification of tears depends on contrast dye passing thru the radiocarpal and distal radial ulnar joint;
- Radiographs:
- may reveal avulsion of ulnar styloid, scaphoid frx, distal radius frx, or there may be volar tilt to the lunate and triquetrum (VISI deformity);
- ulnar variance: (requires zero rotation view);
- variable length of the ulna as a positive or negative variance may influence the carpal position;
- wrist w/ more positive ulnar variance tends to be associated with relatively thinner TFC articular disc;
- MRI:
- normal disc: uniform low signal at both the radial and ulnar attachement;
- complete tear: full thickness discontinuity which extends thru the articular disc;

B: (from Resnick): “a long ulna relative to the length of the radius, designated a positive ulnar variance, may be associated with degenerative changes about the inferior radioulnar joint and ulnar aspect of the radiocarpal joint. A positive ulnar variance may lead to the transfer of excessive compressive forces from the ulna to the triquetrum and lunate via the triangular fibrocartilage complex, to instability at the ulnocarpal joint, and to a condition designated the ulnocarpal impaction (or ulnocarpal abutment) syndrome. The ulnocarpal abutment syndrome may be traumatic (e.g., occurring after Colles' fractures) or nontraumatic, the nontraumatic type increasing in frequency with advancing age of the patient. Radiographic findings include narrowing of the space between the ulna and the carpus and cystic changes, sclerosis, and osteophytosis in apposing portions of the ulna, lunate, and triquetrum. Perforation of the triangular fibrocartilage and disruption of the lunotriquetral interosseous ligament are associated features of this syndrome.”
Please note that A, B, and C can be all be present (from Wheeless' Textbook of Orthopaedics (online)) “…impingement of the ulnar head against the carpi which can lead to TFCC tears and lunotriquetral ligament attrition”, and from Resnick: “Perforation of the triangular fibrocartilage and disruption of the lunotriquetral interosseous ligament are associated features of this syndrome”.

C: Lunotriquetral ligament injuries may result in ulnar-carpal abutment syndrome, but the radiographic finding of lunotriquetral tear (aka dissociation) is described in Wheeless' Textbook of Orthopaedics (online):
- X-rays:
- often radiographs will be unremarkable in lunotriquetral instability;
- PA View:
- shows fully developed volar intercalated-segment instability shows scaphoid to be volar flexed and foreshortened;
- ring sign is present, & distance between the ring & proximal pole of scaphoid is decreased;
- unlike the case with scapholunate dissociations, lunate is volar flexed and triangular;
- triquetrum is dorsiflexed and distal in relation to the hamate;
- on average the ulnar varience is neutral (0);
- Lateral view:
- may occassionally show static palmar flexion of the lunate (VISI deformity)
- normal triquetrolunate angle of approx -16 degrees is converted to neutral or positive angle;
- VISI deformity occurs in only a minority of cases because for it to occur there must be a tear of the dorsal radiocarpal ligament in addition to a tear of the LT interosseous ligament;
- Arthrography:
- may accurately detect triquetral-lunate dissociation in the majority of cases;
- note possible false positive study may be due to age related attritional changes in the LT interosseous membrane;
- w/ a positive study, expect leakage of dye from the radiocarpal to the midcarpal joint or vice versa;

D: Resnick describes reversible ulnar drift with systemic lupus erythematosus, and mentions that the degenerative changes of SLE can be cystic. However, “the deforming nonerosive arthropathy of the hands and wrists (and less commonly the feet) in systemic lupus erythematosus is similar to Jaccoud's arthropathy, which most typically follows rheumatic fever… in the arthropathy of systemic lupus erythematosus, all the digits, including the thumb, frequently are affected, although a more limited distribution may be evident.” Since all the digits are usually involved I think this is a less likely answer.



2. Which of the following is most commonly associated with a fracture involving the growth plate of the phalanx of the great toe?
A. Growth disturbance
B. Subungual exostosis

Best answer: A

A: I’ll assume that this question refers to Salter-Harris type fractures. Prognosis is dependant on the type:
Type I and II: healing is rapid, and growth is rarely disturbed
Type III: prognosis is poor unless there is early accurate reduction
Type IV: even w/ perfect reduction, growth is affected & prognosis is guarded
Type V: growth potential is destroyed
A pictorial review:



B: From Resnick: “A subungual exostosis is an uncommon, benign bone tumor arising in the distal phalanx of a digit, beneath or adjacent to the nailbed. Almost invariably it is a solitary lesion, usually occurring in patients who are in the second or third decade of life and leading to clinical manifestations that include pain, swelling, and ulcerations of the nailbed or surrounding tissue with secondary infection. The great toe is involved in 70 to 80 per cent of cases, followed in order of frequency by involvement of other toes and of fingers (usually the thumb or index finger). Radiographically, the lesion is approximately 1 cm in diameter and projects from the dorsal or dorsomedial aspect of the distal portion of a terminal phalanx (Fig.). It is composed of mature bone with either a broad or narrow attachment to the phalanx. The terminal portion of the exostosis may be flat or cupped and smooth or irregular. Histologically, the lesion consists of a base of trabecular bone with a proliferating fibrocartilaginous cap, features that differ from those of a typical osteochondroma. In the cartilage cap, hypercellularity, frequent mitotic figures, and dark-staining nuclei may be apparent. Although such histologic features may be misinterpreted as evidence of malignant potential, subungual exostoses are uniformly benign and local excision is the treatment of choice. Recurrences are common, however.”



3) A fracture is identified involving the medial aspect of the patella. Most likely etiology for this finding is?
A. quadriceps tendon insertion injury
B. medial meniscus tear
C. lateral dislocation of the patella
D. ACL tear
E. Avulsion from the inferior patellar tendon.

Best answer: C

A: From Resnick: “Partial or complete tears of the quadriceps tendon may result from an injury involving a direct blow to the tendon or forceful flexion of the knee; or they may occur spontaneously in association with such chronic diseases as systemic lupus erythematosus, rheumatoid arthritis, gout, or renal failure, or following corticosteroid therapy. Rarely, spontaneous rupture of the quadriceps tendon occurs in healthy persons. Unilateral or bilateral tendon disruptions have been described. The clinical diagnosis of a complete tear of the quadriceps tendon usually is obvious, based on the patient's history, his or her inability to extend the knee, a hemarthrosis or soft tissue hematoma, and a palpable or visible gap in the soft tissues above the patella”


B: Too much about medial meniscus tear in the books to summarize here. Resnick gives an example of medial meniscus tear in a patient with medial femoral condyle flattening and tibial plateau stress fracture.

C: From Resnick: “Dislocation of the patella is associated with an osteochondral fracture at the medial side of the patella and, less frequently, at the lateral margin of the lateral femoral condyle. Although this diagnosis is obvious if the patient is seen when the patella is dislocated, the patella may become reduced spontaneously, and the patient may not volunteer a history that is appropriate for the injury.”


D: From Wheeler: “A small avulsion frx of lateral tibial condyle just below joint line is now recognized as a sign of injury of ACL.” (Figure). I could not find anything in Resnick to corroborate this statement. Resnick mentions associations of ACL tears and lateral patellar dislocations (which are associated with medial patellar fractures). Resnick also mentions associations with osteochondral fracture of the lateral femoral condyle.



4. A 58-year-old female presents with chronic foot pain, a pes planus deformity, as well as periosteal reaction along the distal aspect of the medial malleolus on plain film. The most likely diagnosis is:
A. charcot joint
B. Plantar fascitis
C. Posterior tibial tendon rupture
D. Peroneus longus tendon rupture
E. Stress fracture

Best answer: C

A: Charcot joints are described as a bag of bones. They aquire the “rockerbottom foot” (figure) and usually require special orthotics.


B: Wheeler states: “no definitive evidence that plantar fasciitis is linked with abnormal foot posture (pes planus or pes cavus)”, and describes the radiographic finding:
- X-rays:
- lateral view:
- "saddle sign" is present in 60 % of cases of heel pain: radiolucency proximal to plantar calcaneal spur;
- may demonstrate spurring of the medial calcaneal tuberosity

C: From Resnick: “Spontaneous Achilles tendon rupture has been described in patients with rheumatoid arthritis, some of whom have received local corticosteroid injections. These ruptures may have resulted from collagenolysis of the tendon as a result of its proximity to an inflamed retrocalcaneal bursa. Tendon inflammation itself or enthesitis may be a contributing factor in other patients with this complication. Such tendon rupture appears to be more frequent in the hands and the wrists, although tenosynovitis, especially of the peroneal and posterior tibial tendons, is a well-known feature of the disease, and rupture of the last of these tendons may lead to flatfoot deformity.”
Posterior Tibial tendon (from Snell): “It plays an important part in holding up the medial and lateral longitudinal arch in the foot”

D: Resnick: “Displacement of the os peroneum may occur in patients with rupture of the tendon of the peroneus longus muscle. This sesamoid bone ossifies in approximately 5 to 20 per cent of persons, and normally is located adjacent to the lower border of the cuboid bone or at the level of the calcaneocuboid joint. Proximal migration with or without fragmentation of the os peroneum is consistent with attrition and rupture of the peroneus longus tendon just distal to the location of the sesamoid bone”. Also, the peroneus longus tendon runs near the lateral malleolus, so the medial malleolus should not be involved in isolated injury.

E: Stress fracture of the medial malleolus could not explain a pes planus deformity.


5. The most common sites of tarsal coalition are:
calcaneonavicular joint and middle facet of
calcaneonavicular joint and posterior facet of
talonavicular joint and middle facet of
talonavicular joint and posterior facet of

Best answer: A?

I could find no reference that mentioned the middle and posterior facets of the calcaneonavicular joint, so the question might not be remembered correctly. From Resnick: “Tarsal coalition represents an abnormal fusion of one or more of the tarsalia. The union may be fibrous, cartilaginous, or osseous and can be congenital (developmental) or acquired in response to infection, trauma, articular disorders, or surgery… Isolated partial coalitions can be classified according to the bones that are affected; calcaneonavicular, talocalcaneal, talonavicular, and calcaneocuboid fusions, in order of decreasing frequency, can be detected.” Calcaneonavicular bony bridges can be seen on the lateral view with the classic "anteater nose" coming from the calcaneous.







6. Most common site of post-traumatic osteolysis:
A. hip
B. clavicle
C. pubis
D. humerus

Best answer: B

Resnick section on post traumatic osteolysis speaks almost exclusively about the clavicle, so I’ll assume it’s the most common: “Although some degree of bone loss is common after traumatic insult, particularly when complicated by fracture, in certain situations and sites the degree of posttraumatic osteolysis may appear excessive. Knowledge of the existence and appearance of such osteolysis is important so that the resorption of bone that accompanies a traumatic insult is not mistaken for an inflammatory or neoplastic process…Posttraumatic osteolysis can lead to progressive resorption of the outer end of the clavicle…Prominent posttraumatic osteolysis also has been noted in the ulna, radius, and carpal bones, In the femoral neck, resorption and rotation at a fracture site can produce a radiographic picture that may be misinterpreted as a malignant process. Osteolysis after odontoid fracture or atlantoaxial subluxation can produce a separate bone at the tip of the dens, resembling or identical to the os odontoideum. Other sites of posttraumatic osteolysis are the skull, sternum, and ribs.”



7. Atlantoaxial subluxation is NOT associated with which of the following?
A. Down’s syndrome
B. Juvenile rheumatoid arthritis
C. Trauma
D. Hypothyroidism
E. Spondyloepiphyseal Dysplasia

Best answer: D

From Resnick: “Radiographic abnormalities of the cervical spine are a significant feature of juvenile chronic arthritis, although neurologic alterations resulting from such cervical spine abnormalities are less frequent than in adult-onset rheumatoid arthritis. Subluxation may develop in any vertebral segment but is most characteristic at the atlantoaxial level. Atlantoaxial subluxation (greater than 4 or 5 mm between the posterior surface of the anterior arch of the first cervical vertebra and the anterior surface of the odontoid process) can be observed in juvenile-onset ankylosing spondylitis and Still's disease but is most characteristic of juvenile-onset adult type (seropositive) rheumatoid arthritis. It should be remembered that atlantoaxial instability in the child is not diagnostic of juvenile chronic arthritis, being observed in trauma and a variety of conditions including those, such as Down's syndrome, with hypoplasia of the odontoid process and congenital weakening of the surrounding ligaments, and those associated with inflammation in the neck. The last-mentioned situation frequently is called Grisel's syndrome; in this disorder nontraumatic atlantoaxial subluxation accompanies peripharyngeal inflammatory disease. The cause of this syndrome is unknown, but the recent demonstration of vascular continuity between the pharyngovertebral veins and the periodontoidal venous plexus and suboccipital epidural sinuses suggests that hematogenous spread of infection is a pathogenetic factor.”
Regarding Spondyloepiphyseal Dysplasia; “This short-trunk dwarfism is distinguished by mild shortening of the limbs, flat face, cleft palate, short neck, increased anteroposterior chest diameter, and joint restriction. During growth, progressive kyphoscoliosis, dorsal kyphosis, or lumbar lordosis occurs. The hands and feet often are normal except for the presence of equinovarus deformity. Important additional features include myopia and retinal detachment, which can lead to blindness, and atlantoaxial instability, which can result in spinal cord compression that can be shown nicely by MR imaging. Although the pattern of inheritance usually is autosomal dominant, an autosomal recessive form of the disorder also is probable.”

I could find no association between hypothyroidism and atlantoaxial subluxation.


8. Which is true regarding nursemaid’s elbow ?
A. subluxation of the radial head beyond the annular fibers
B. dislocation of the radial-carpal joint???

Best answer: A

A: From Resnick: “In the child, subluxation of the radial head, which usually but not invariably is transient in nature, is termed nursemaid's elbow or pulled elbow. It is produced by a sudden pull on the child's arm which, when the forearm is in a pronated position, results in slipping of the radial head beneath the annular ligament as it is torn from its attachment to the radial neck... The initial radiographic diagnosis is based on an abnormal position of the radial head, which usually is displaced anteriorly”

B: This answer was probably not remembered correctly.



9. In a patient with post-traumatic collapse of the vertebral bodies, plain films may reveal
A. increasing density/progressive sclerosis of the vertebral body
B. involvement of the intervertebral disc
C. osteophyte formation

Best answer: none of the above.

Post-traumatic collapse (aka post-traumatic osteonecrosis) of vertebral bodies is also called Kümmell's disease. According to Resnick: “Kümmell's disease is a form of delayed, posttraumatic collapse of the vertebral body. In 1891, Kümmell reported a delayed posttraumatic vertebral collapse that occurred weeks or even months after an injury. Many underlying mechanisms, including traumatic, nutritional, vasomotor, and neurologic, have been suggested as causative factors. At present, the most widely accepted mechanism for Kümmell disease is osteonecrosis. In 1978, Maldague and coworkers noted the association between vertebral body ischemia and the presence of gas within the vertebral body, also known as the intravertebral vacuum cleft. These authors suggested that the basis for Kümmell disease may be ischemia associated with the episode of trauma, which leads to osteonecrosis and delayed collapse of the vertebral body. Although Kümmell himself did not note the vacuum cleft sign, evidence exists that osteonecrosis probably is the mechanism accounting for delayed posttraumatic vertebral collapse. Almost all reported cases of intravertebral vacuum clefts occur in noninfectious and non-neoplastic conditions.
The typical vacuum cleft appears as a transverse radiolucent line seen in the centrum of the collapsed vertebral body or adjacent to one of its endplates. The vacuum cleft is exaggerated by spinal extension, particularly in the lateral decubitus position, and diminishes with spinal flexion and prolonged supine positioning. On CT scans, the gas collection may appear more inhomogeneous and irregular than it does on plain films.”





10. A 35-year-old woman has a sclerotic lesion in the humerus that blends into the margins. Bone scan demonstrates slightly increased uptake. The most likely diagnosis is:
A. osteoblastoma
B. osteoid osteoma
C. bone island

Tough question; either A or B. Bone islands are typically “cold” in nuclear medicine, osteoblastomas and osteoid osteomas are “hot”, so exlude C. Some literature considers osteoblastomas as just a big osteoid osteoma. Both may have sclerotic boarders. Age (adolescent to young adult) and sex (female) predilection don’t clearly favor either. Location was not a big help either… long bones are involved in 30% of osteoblastomas and 70% of osteoid osteomas. I could find no information regarding the incidence of osteoblastomas verses osteoid osteomas, but Resnick sites 298 published osteoblastoma cases and over 1000 osteoid osteoma cases. Osteoblastomas are larger than osteoid osteomas and therefore would be more conspicuous on plain films. If the sclerotic lesion is detected by plain films, I would choose osteoblastoma because it is larger/more conspicuous. If the sclerotic lesion is detected by CT, I would choose osteoid osteoma because it is more common.

A: Resnick: “An osteoblastoma may have a subperiosteal, cortical, or medullary location. Although cortical or medullary tumors are most frequent, subperiosteal osteoblastomas are not uncommon in the facial and cranial bones, especially the maxilla, and also occur in the humerus, radius, ribs, femur, and tibia. Intracortical osteoblastomas are associated with a marked amount of surrounding sclerotic bone similar to that found in an osteoid osteoma, but the nidus of an osteoblastoma is larger. Because the periosteum apparently is required for the production of sclerotic bone, osteoblastomas located in the spongiosa lack abundant osteosclerosis.” Also remember that osteoblastomas may look like aneurismal bone cysts, FEGNOMASHIC.

B: Resnick “The term osteoid osteoma was introduced by Jaffe 12 in 1935 to describe a benign osteoblastic tumor with distinctive histologic abnormalities consisting of a central core of vascular osteoid tissue and a peripheral zone of sclerotic bone. Subsequent descriptions of this lesion have been numerous, with in excess of 1000 cases being reported in the literature, firmly establishing the osteoid osteoma as one of the more frequent and characteristic tumors or tumor-like lesions of bone… An osteoid osteoma is a lesion that characteristically is quite “hot” on the bone scan and yet may be difficult to detect on a radiograph.”

C: Resnick: “Enostoses, or bone islands, appear radiographically as single or multiple, ovoid, round, or oblong, intraosseous sclerotic areas with discrete margins and thorny, radiating spicules. Of diagnostic help in such cases, bone scintigraphy almost uniformly is negative in instances of enostoses.”


11. Ganglion cyst in the shoulder (not remembered well):
A. related to a labral tear
B. associated with atrophy of the supraspinatus
C. rotator cuff tear

Best answer: A

This requires some knowledge of suprascapular nerve anatomy ala. Resnick: “The suprascapular nerve is a mixed motor and sensory nerve that carries pain fibers from the glenohumeral and acromioclavicular joints and provides motor supply to the supraspinatus and infraspinatus muscles. It is derived from the upper trunk of the brachial plexus, originating from the fourth, fifth, and sixth cervical nerve roots. The suprascapular nerve passes deep to the trapezius and omohyoid muscles to enter the supraspinatus fossa, passing beneath the superior transverse scapular ligament; it then runs deep to the supraspinatus muscle to enter the infraspinatus fossa through the spinoglenoid notch, passing beneath the inferior transverse scapular ligament at the lateral margin of the spine of the scapula. Two motor branches supplying the supraspinatus muscle are derived from that portion of the suprascapular nerve that is in the supraspinatus fossa and, similarly, two motor branches supplying the infraspinatus muscle are derived from that portion of the suprascapular nerve that is in the infraspinatus fossa. Therefore, depending upon the precise site of involvement, entrapment of the suprascapular nerve can lead to weakness and atrophy of both the supraspinatus and infraspinatus muscles, or to weakness and atrophy of the infraspinatus muscle alone.”


A: From Resnick: “Certain tears of the anterior or posterior portion of the labrum are accompanied by the development of perilabral cysts or ganglia. The pathogenesis of the cystic lesions is not certain, although similar lesions have been described in association with meniscal tears of the knee (where they are designated meniscal cysts) and with tears or degeneration of the acetabular labrum (in which case they are termed ganglion cysts). In each of these locations, fluid derived from the joint might extend through the tear of the labrum or meniscus into the surrounding soft tissues, leading to cyst formation. In the shoulder, such perilabral cysts or ganglia may extend into the spinoglenoid notch or the suprascapular notch of the scapula, or both notches, and produce an entrapment neuropathy.”

B: From Resnick: “entrapment of the suprascapular nerve can lead to weakness and atrophy of both the supraspinatus and infraspinatus muscles, or to weakness and atrophy of the infraspinatus muscle alone. Ganglia arising near the spinoglenoid notch affect a more distal segment of the nerve, resulting in selective involvement of the infraspinatus muscle.”

C: not related.

12. What is the most common x-ray finding of failed hip prosthesis?
A. loosening
B. infection

Best answer: A

From Resnick: “Loosening is the most common cause of failure of cemented prostheses. It may be due to inadequate component fixation or to infection. Loosening rates differ in the various reported series, and comparison is made difficult by the lack of uniformity in its definition. Improvements in cement technique have reduced loosening rates on the femoral side. In contrast, acetabular loosening has been found to increase with time, and efforts at improving cement fixation have been less successful. Mechanical loosening has been reported in percent of cases.”



13. Which muscle of the rotator cuff inserts on the lesser tuberosity?

A. supraspinatus
B. infraspinatus
C. teres minor
D. subscapularis

Best answer: D

Muscle Origin Insertion Action Innervation Artery

subscapularis medial two-thirds of the costal surface of the scapula (subscapular fossa) lesser tubercle of the humerus medially rotates the arm; assists extention of the arm upper and lower subscapular nerves (C5,6) subscapular a.
supraspinatus supraspinatous fossa greater tubercle of the humerus (highest facet) abducts the arm (initiates abduction) suprascapular nerve (C5,6) from the superior trunk of the brachial plexus suprascapular a.
teres minor upper 2/3 of the lateral border of the scapula greater tubercle of the humerus (lowest facet) laterally rotates the arm axillary nerve (C5,6) from the posterior cord of the brachial plexus circumflex scapular a.
infraspinatus infraspinatous fossa greater tubercle of the humerus (middle facet) laterally rotates the arm suprascapular nerve suprascapular a.



14. Which of the following comprise the pes anserinus?
A. sartorius, gracilis, rectus femoris
B. sartorius, gracilis, semitendinosis
C. sartorius, semitendinosus, semimembranosis
D. gracilis, semitendinosus, semimembranosis

Best answer: B

The distal tendons of the sartorius, gracilis and semitendinosus pass medial to the medial collateral ligament on their way to the tibial tuberosity. These tendons are collectively known as the pes anserinus (Latin for foot of the goose), because of their shape. A bursa lies between these three tendons and the MCL, known, oddly enough, as the anserine bursa. Occasionally, one can see a fluid collection in this bursa due to anserine bursitis. This should not be mistaken for a popliteal cyst, which occurs more posteriorly, between the tendon of the medial head of the gastrocnemius and the semimembranosus tendon.




15. Tumor-related osteomalacia is seen with:
A. chondroblastoma
B. non-ossifying fibroma
C. fibrous dysplasia
D. hemangiopericytoma

Best answer: D

From Resnick: “Hypophosphatemic vitamin D-refractory rickets and osteomalacia in association with various neoplasms have been recognized with increasing frequency since Prader and coworkers first description in 1959. In 1976, Linovitz and associates 214 reviewed 11 cases and added two new patients. Since then, additional reports have appeared. The associated neoplasms have occurred in children and adults, are located in soft tissues or bone, and vary in size. The lesions typically are vascular and often show foci of new bone formation; the most frequent histologic diagnosis has been hemangiopericytoma. Although most of the neoplasms have been of mesenchymal origin, the syndrome has been reported in association with prostatic carcinoma and with oat cell carcinoma of the lung. Bone lesions have included nonossifying fibroma, giant cell tumor, osteoblastoma, and non-neoplastic diseases such as fibrous dysplasia and neurofibromatosis.”


16. The triangular-fibrocartilage separates which compartments:
A. radiocarpal and midcarpal
B. common carpometacarpal and radiocarpal
C. first carpometacarpal and radius
D. distal radius/ulna and radiocarpal

Best answer: D

D: From Resnick: “In recent years, great interest has developed regarding the technique of three-compartment wrist arthrography, in which separate injections into the radiocarpal, midcarpal, and inferior radioulnar compartments are employed. With injuries to the triangular fibrocartilage or ulnar styloid, communication between the radiocarpal and inferior radioulnar compartments is observed.”

For those of us (like myself) who have never performed wrist arthrograms, here are normal examples of the three compartments:

Normal radiocarpal joint.
Normal midcarpal joint. Normal distal radioulnar joint
triangularfibrocartilage tear
Orange arrow = radiocarpal joint
Blue arrow = volar radial recesses
Green arrow = opacification of an extensor tendon sheath
White arrow = prestyloid recess
Yellow arrow = abnormal opacification of the inferior (distal) radioulnar joint
Red arrow = abnormal leakage of contrast possibly due to a capsular tear


17. Question regarding associations of coned epiphysis.

Resnick has little to say about coned epiphysis: “These are particularly frequent in the hands and feet, although they may occur elsewhere, including the long tubular bones. Cone-shaped epiphyses and inverted V, “cup,” or “channel” deformities of the adjacent metaphyses are observed in sickle cell anemia, although similar changes occur in many other disorders, including congenital diseases, infection, trauma, and radiation injury…Osteoglophonic dysplasia, a severe form of rhizomelic dwarfism, is characterized by facial deformities, including a flat bridge of the nose, frontal bossing, hypertelorism, and enlargement of the jaw. Radiographic findings include craniostenosis, prognathism, cystic lesions in the jaws, mild platyspondyly, multiple metaphyseal radiolucent areas that are most frequent in the distal portions of the femora, and epiphyseal alterations.”

On the Internet I found various other associations:
brachydactyly,
Dyggve-Melchior-Clausen syndrome
Trichorhinophalangeal Syndrome Type II (TRPS2, also known as Langer-Giedion Syndrome)


18.

Answer: C

Superior vena cava syndrome is obstruction of SVC with development of collateral pathways. Sxs: head and neck edema (70%), cutaneous enlarged venous collaterals, headache, dizziness, syncope, proptosis, tearing, dyspnea, cyanosis, chest pain, hematemesis. See superior mediastinal widening, SVC thrombus, occlusion, or compression, dilated cervical and superficial thoracic veins.
Etiology: Malignant lesion (80-90%) – rapid progression
1. Bronchogenic Ca (>50%) – Small cell is most common type
2. Lymphoma
Benign lesions – slower onset
1. Granulomatous mediastinitis (histo, sarcoid, TB)
2. Substernal goiter
3. Ascending aortic aneurysm
4. Pacer wires/CVC (23%)
5. Constrictive pericarditis
Bronchogenic Carcinoma:
1. Small cell undifferentiated ca (15%)
 Strong association with cigs
 Centrally located, highly malignant
 Most common cell type assoc. with superior vena cava obstruction
 Ectopic hormone production: SIADH, Cushing syndrome
2. Squamous cell ca (30-35)
 Strong assoc. with cigs
 Centrally located, lowest incidence of mets
 Most common cell type to cavitate
 Most common cell type that is radiologically occult
 Most common cell type to cause Pancoast tumor
 Ectopic hormone production: parathyroid hormone-like substance
3. Adenocarcinoma (50%)
 Most common cell type seen in women and nonsmokers
 Peripheral, upper lobe distribution, intermediate malignant potential
 Frequently found in scars (TB, infarct, scleroderma, bronchiectasis)
 Also a frequent cause of Pancoast tumor
 Bronchioloalveolar carcinoma is subtype (air bronchograms)
4. Undifferentiated large cell carcinoma (<5%)
 Strong assoc. with smoking
 Large bulky usually peripheral mass with intermediate malignant potential
 Giant cell carcinoma is subtype (very aggressive, poor prognosis)



19.

Answer: A is best. I hate internal medicine questions, damn it!

Complications assoc. with MI include: left ventricular failure (60-70%), aneurysm (12-15%), myocardial rupture (3.3%), rupture of papillary muscle (1%), rupture of interventricular septum (0.5-2%), and Dressler syndrome (post MI syndrome).
In pts. that develop LV failure, pulmonary edema is associated with 44% mortality.
Signs of pulmonary venous hypertension are a good predictor of mortality (>30% if present, <10% if absent).
Cardiogenic shock (systolic pressure <90mm Hg) is assoc. with 80-100% mortality.
Rupture of papillary muscle results in massive mitral insufficiency with abrupt onset of severe persistent pulmonary edema. There is only minimal LV enlargement and NO dilatation of the LA. Mortality is 70% within 24 hrs and 80-90% within 2 weeks.
Myocardial rupture results in almost 100% mortality and occurs 3rd to 5th day post MI. See enlargement of the heart (hemopericardium).
Rupture of interventricular septum occurs within 4-21 days. Results in L to R shunt, right-sided cardiac enlargement, engorgement of pulmonary vasculature, and NO pulmonary edema. Mortality is 24% within 24 hrs, 87% within 2 months, >90% in 1 year.
Dressler syndrome is pericarditis, pleuritis, and pneumonitis. Occurs 2-3 wks (range 1 wk to several months) following MI (or surgery). Fever is present. Autoimmune reaction. May relapse as late as 2 years after initial episode.



20. A patient with laryngeal papillomatosis is at greatest risk for which of the following?
A. Adenocarcinoma
B. squamous cell carcinoma
C. lymphoma
D. other cancers

Answer: B

Squamous papilloma is the most common benign tumor of the larynx. Malignant transformation is into invasive squamous cell carcinoma.
Laryngeal papillomatosis: HPV types 6 and 11. Bimodal distribution. <10 yrs (juvenile papillomatosis) diffuse involvement. Probable transmission from mother to child during vaginal delivery. 21-50 yrs usually single papilloma.
Location: 1. uvula, palate. 2. vocal cord. 3. subglottic extension (50-70%). 4. pulmonary involvement (1-6%) – tracheobronchial and pulmonary papillomatosis.

Dahnert p. 326

21. Which of the following is not a cause of unilateral rib notching?
A. Blalock-Taussig shunt
B. Aberrant subclavian artery in a patient with coarctation
C. Coarctation which is located between the common carotid arteries
D. Brachial artery stenosis
E. Coarctation between innominate and left carotid arteries

Answer: D

Causes of unilateral inferior rib notching: Coarctation proximal to left subclavian or aberrant right subclavian artery, subclavian artery occlusion, Blalock-Taussig shunt, neurofibromas.
Causes of bilateral inferior rib notching: Typical coarctation, aortic thrombosis, Takayasu’s aortitis, CHD with decreased pulmonary flow, SVC obstruction.

I could not find a reference which listed brachial artery stenosis as a cause of rib notching. This is probably too distal to result in rib notching. Subclavian artery occlusion, however, is a cause of unilateral rib notching.

For choices C and E, I could not find a reference which describes coarctation occurring in these locations, however, any aortic narrowing which occurs between the right and left subclavian arteries would eventually produce unilateral rib notching (see below).

Blalock-Taussig shunt is a known cause of unilateral rib notching. This procedure is usually performed for Tetralogy of Fallot, and is basically an anastamosis between the subclavian artery and pulmonary artery on the opposite side of the aortic arch. Since blood is diverted from the subclavian artery, prominent collaterals will cause rib notching on the side of the anastamosis.

Classically, coarctation is of two types: diffuse (tubular, infants) and localized (post-ductal, adults). Localized is most common type. Usually occurs close to ligamentum arteriosum, distal to left subclavian artery. This will produce bilateral rib notching between ribs 3-8 (most prominent at ribs 3 and 4). Coarctation is the most common cause of bilateral inferior rib notching. This is a result of increased blood flow through intercostal arteries to circumvent obstruction. Erosion of the inferior rib margin occurs over time. Notching is rare under the age of 7 (usually adults over age 20). The first two ribs are uninvolved since they arise from the superior intercostal branch of the costocervical trunk of the subclavian artery (proximal to level of obstruction).
Coarctation may produce unilateral left rib notching when the aortic narrowing occurs proximal to an aberrant right subclavian artery. Conversely, when the coarctation is proximal to the left subclavian artery, unilateral right rib notching occurs. Collateral pathways are formed on the side of the “patent” subclavian artery.

Often present with lower extremity hypotension relative to upper.
See figure 3 sign (result of pre and post stenotic bulging of aorta) and reverse figure 3 sign (barium swallow).

Coexistent cardiac anomalies are uncommon with localized type, which is usually incidental finding late in life.
Infantile (diffuse, tubular) coarctation is hypoplasia of a long segment of aortic arch after origin of innominate artery and is commonly assoc. with coexistent cardiac anomalies. Most commonly, bicuspid aortic valve (25-50% or 80% depending on source). May also be assoc. with PDA, VSD, TEF, and Circle of Willis aneurysms among many others. Neonatal CHF is seen in 50% (Second most common cause after hypoplastic left heart). Mortality is 11% prior to 6 months of age. Surgical correction usually at 3-5 yrs of age.

Dahnert, Pedi Req., Brant & Helms

22. A patient with total anomalous pulmonary venous return has a chest radiograph which reveals a snowman or figure of eight configuration to the mediastinum. The structure which accounts for the superior left mediastinal border is:
A. azygous vein
B. aortic arch
C. vertical vein
D. left superior pulmonary vein
E. innominate vein

Answer: C

TAPVR = saturated blood from pulmonary veins returns to right side of heart (a closed circuit with no communication to left side of heart; this is incompatible with life without a R-L communication such as ASD).
Incidence 2%.
R to L shunt of admixed blood occurs through large ASD.
TAPVR highly assoc. with asplenia.
PAPVR (more commonly) and TAPVR are assoc. with polysplenia.

In TAPVR, the snowman or figure of eight configuration of the mediastinum is created by a distended SVC (right border forming structure) and a prominent left vertical vein (left border forming structure which is a persistent left SVC). This is seen in Type 1 TAPVR (supracardiac). This is most common type (55%). The pulmonary veins drain cephalad into the left vertical vein, into left BCV, then into SVC or occasionally the azygos.

Type 2 TAPVR = cardiac (30%). Pulmonary veins drain into coronary sinus or directly into RA. Non-specific appearance (like any L to R shunt). Rarely obstructive.

Type 3 TAPVR = infracardiac (12%). Common pulmonary vein descends thru esophageal hiatus to drain into portal vein/ductus venosus, hepatic vein, or IVC. Pulmonary venous return is always obstructed and, therefore, results in pulmonary edema. Distinctive appearance, normal sized heart with pulmonary edema.
Cyanotic pulmonary edema in 1st week of life = TAPVR type 3.

Type 4 TAPVR = mixed (5%).

All types are over circulated and cyanotic. Obstruction occurs in all type 3 and in 25 – 40% of type 1 and 4, which causes pulmonary edema.

Partial APVR (PAPVR; clinically insignificant). Most commonly is return of RUL pulm. Vein directly into SVC (assoc. with sinus venosus ASD).
Scimitar syndrome is PAPVR usually in R lung base; it drains into IVC. Assoc. with ipsilateral pulmonary hypoplasia.

AFIP notes

23. Which of the following has a vascular ring?
A. Right aortic arch with an aberrant left subclavian artery
B. Left aortic arch with an aberrant right subclavian artery
C. Left pulmonary artery which arises posteriorly from the right pulmonary artery
D. Mirror image right aortic arch

Answer: A

True vascular rings cause concomitant posterior esophageal and anterior tracheal impressions. The most common entity is a double aortic arch or a right aortic arch combined with an aberrant left subclavian artery. The ring is completed by the ductus remnant. “Right aortic arch with an aberrant left subclavian artery is the second most common type of vascular ring after a double aortic arch (55%).” These two entities are indistinguishable by esophogram (when dominant arch is on right).

True vascular rings completely encircle the trachea and esophagus and are usually symptomatic. These include:
1. Double aortic arch with R descending aorta and L ductus (55%).
2. R aortic arch with R descending aorta, aberrant L subclavian, and L ductus.
3. L arch with L descending aorta, aberrant R subclavian, and R ductus (extremely rare).

Pulmonary sling = Aberrant left pulmonary artery (L pulm artery arises posteriorly from right pulm artery). The aberrant L pulm artery usually passes between the trachea and esophagus and, thus, creates an anterior impression on the esophagus and posterior indentation on the trachea. Usually symptomatic.

Mirror image right aortic arch is the second most common aortic arch anomaly behind L arch with aberrant R subclavian. Type 1 is most common and is associated with cyanotic congenital heart disease in 98% of cases (Tet most commonly - 87%). 25% of patients with tetralogy have a R arch and 37% of patients with truncus arteriosus have a R arch (Tet is much more common, however. i.e. truncus more highly assoc. with a R arch, but if have a R arch more likely to see tetralogy). No vascular ring, no retroesophageal component, no structure posterior to trachea. Type 2 is very rare, but does result in a true vascular ring.

Left arch/aberrant right subclavian artery is the most common arch anomaly, being found in 1% (0.4 – 2.3%) of individuals. The right subclavian artery arises as a fourth branch of the aortic arch and must cross the mediastinum to reach the right arm. It crosses behind the esophagus in 80% of cases, between the trachea and the esophagus in 15%, and anterior to the trachea in 5%. It is associated with congenital heart disease in 10% of patients. Seen in 37% of Down syndrome children with CHD. Usually asymptomatic (not a true vascular ring).

A similar question from 1998 was answered incorrectly (very tricky question). The question asks “What is the most common vascular ring?” The answer should be (from choices given) R arch with aberrant L subclavian (for reasons stated above). It is true that L arch with aberrant R subclavian is much more common. This, however, is not usually a true vascular ring. To form a true vascular ring, there would have to be a R sided ductus, which is very rare.

If this question read, “Which of the following does not have a vascular ring?” the best choice would be D. If the question read, “Which of the following is the most common arch anomaly?” the answer would be B.

Dahnert, Pedi Req, Brant & Helms

24. Which of the following is not a typical radiographic finding in a patient with rheumatoid arthritis?
A. Pleural effusions
B. Cavitating nodules
C. Adenopathy
D. Basilar fibrosis

Answer: C

No adenopathy!
Pulmonary manifestations of Rheumatoid arthritis:
 Pleural effusion, mostly unilateral, without change for months, usually not associated with parenchymal disease
 Interstitial fibrosis – lower lobe predominance
 Rheumatoid nodules (30%): well-circumscribed, peripheral, frequent cavitation.
 Caplan syndrome = hyperimmune reactivity to silica inhalation with rapidly developing multiple pulmonary nodules.
 Pulmonary htn secondary to arteritis.

Dahnert

25. All of the following are recognized causes of pulmonary arterial hypertension except:
A. emphysema
B. schistosomiasis
C. pulmonary embolism
D. Eisenmenger physiology
E. Some other choice, mitral valve problem

Answer: D

Emphysema, schistosomiasis, and PE are all possible causes of pulmonary arterial hypertension.
Schistosomiasis is caused by three blood flukes. Human infestation acquired through contact with infested water. Mature flukes (in mesenteric or bladder venules) produce ova, which may embolize to the lungs where they implant in and around small pulmonary arterioles. The organism induces granulomatous inflammation and fibrosis, which leads to an obliterative arteriolitis resulting in pulmonary hypertension and cor pulmonale.
Schistosomiasis is also a common cause around the world of portal venous hypertension.
Eisenmenger physiology is a result of pulmonary arterial hypertension, not a cause. With increased pulmonary blood flow (L to R) shunt, damage to pulmonary arteries results in increased pulmonary vascular resistance and pulmonary arterial hypertension. This may eventually lead to reversal of the shunt to R to L, which is termed Eisenmenger physiology.

Brant & Helms

26. Which nerve passes beneath the aortic arch at the aorticopulmonic window?
A. left vagus nerve
B. right vagus nerve
C. left recurrent laryngeal nerve
D. right recurrent laryngeal nerve
E. left phrenic nerve

Answer: C

The recurrent laryngeal nerve is a distal vagal branch which provides motor function to the inferior constrictor muscle and endolaryngeal muscles (vocal cord function). The Vagus nerve (X) exits the cranium at the jugular foramen and plunges along the posterolateral aspect of the carotid artery to the AP window on the left and the clavicle on the right. The right recurrent laryngeal turns cephalad around the right subclavian artery, whereas the left recurrent laryngeal turns cephalad by looping through the AP window. Both ascend to reach the larynx in the tracheoesophageal groove.

The phrenic nerve innervates the diaphragm (C3,4,5).

Harnsberger

27. Anterior motion of the mitral valve during systole is associated with which of the following?
A. mitral stenosis
B. idiopathic hypertrophic subaortic stenosis (IHSS)
C. mitral valve prolapse
D. atrial myxoma
E. ruptured papillary muscle

Answer: B

Systolic anterior motion (SAM) = idiopathic hypertrophic subaortic stenosis (IHSS) aka hypertrophic obstructive cardiomyopathy (HOCM).

Diastolic anterior motion = normal, left atrial myxoma

Systolic posterior motion = mitral valve prolapse, ruptured papillary muscle.

Decreased diastolic anterior motion = mitral stenosis.

This question involves assessing the motion of the mitral valves in different cardiac diseases.
Normally, the mitral valves would have anterior motion in diastole when the LA contracts  forcing blood into the LV

Mitral prolapse (MP) is one of many etiologies which results in mitral regurgitation (MR). Mitral regurgitation is defined as LV blood regurges  LA in systole when the LV contracts. Aside from MP, other causes of MR includes: abnormality to the chordae, papillary muscle, mitral annulus, and adjacent LV wall. The hallmark of MP  posterior displacement of mitral leaflets behind the mitral annulus LA during systole.
The degree of MR thus depends on the severity of the prolapse.

Papillary rupture (PR) usually results from acute MI. Other rare causes include: chest trauma, endocarditis. Since each papillary muscle supplies chordae tendinae to both mitral leaflets (There are 2 leaflets: posterior & anterior), rupture of papillary muscle produces severe MR. Motion of the mitral leaflets range from moderate prolapse to complete flailing. Therefore the answer is the same as in MP.

IHSS (Idiopathic Hypertrophic Subaortic Stenosis) is a dynamic form of subaortic stenosis. The obstruction occurs during systole. In systole, the anterior leaflet of the mitral valve moves into the aortic outflow region causing occlusion of LV outflow. This occurs due to rapid blood flow through the narrow outflow tract which creates a Venturi effect, pulling the anterior leaflet of the mitral valve anteriorly. During diastole, the stenosis disappears, the mitral valve resumes its normal position.

Atrial myxoma is the most common primary tumor of the heart. The tumor frequently arises from the left side of the fossa ovale but it also may arise anywhere in the atrium. It may be sessile or pedunculated. It may be single or multiple. Echocardiogram usually shows a discrete homogeneous mass. When large and pedunculated, the AM may prolapse across the mitral valve in diastole and impair LV filling.

Most mitral stenoses (MS) are acquired from rheumatic carditis which occurred 5-10 years earlier. The hallmark of MS is calcified mitral valves, hypokinetic valve motion, and domed mitral leaflets. During diastole, there is decreased anterior motion of the leaflets.

Cardiac the Requesites, Dahnert

28. What is the most likely cause of left ventricular failure in a patient with emphysema?
A. pulmonary artery hypertension
B. atherosclerotic disease
C. Eisenmenger’s syndrome

Answer: B

B is the best choice (process of elimination), although I could not find a source which specifically stated this to be the case.
Pulmonary arterial hypertension may occur in patients with severe, long-standing emphysema and can result in right heart failure (not left)  cor pulmonale (maybe the question is supposed to refer to most common cause of right heart failure).
Eisenmenger’s syndrome does not exist in patients with emphysema. This occurs in patients that develop PAH from long standing L to R shunt, which then reverses to R to L shunt.

29. Area with the most increased/highest increase in pulmonary vascular resistance:
A. Segmental arteries
B. Subsegmental arteries
C. Muscular branches
D. Capillaries

Answer: D?

I could not find a source which directly answered this question and I’m not exactly sure how to interpret it. This said, I would choose D since resistance is proportional to 1/(radius)4.

30. A 45 yr old male has flu-like symptoms for 1-2 weeks. Chest x-ray demonstrates mediastinal adenopathy. The most likely diagnosis is:
A. Klebsiella
B. E. Coli
C. Mycoplasma
D. Burkitt’s lymphoma

Answer: C?

All of these choices suck. If I had to pick one of these, I would go with Mycoplasma since it is the most common and the symptoms fit. Mycoplasma is sometimes associated with hilar adenopathy but I couldn’t find any particular association with mediastinal adenopathy.

NHL would be a great choice – maybe this was one of the choices rather than Burkitt lymphoma, which would be extremely unlikely.

Mediastinal adenopathy is not characteristically associated with Klebsiella, E. Coli, or Mycoplasma infection. E. Coli would be very rare.

Burkitt Lymphoma – most common type of NHL in children; initially described in Africa. Tumor from undifferentiated B-cell derived lymphocytes; associated with EBV. Seen in children and young adults. Location – mandible and maxilla; multifocal (10%).
 Jaw mass
 Abdominal mass
 Paraplegia
 No peripheral leukemia
 Usually intraabdominal extranodal involvement with sparing of spleen.
Endemic form seen in areas with malaria – tropical Africa, New Guinea. Age is 6-8 yrs. Nonendemic form – age is 10-12 yrs. Usually see abdominal involvement. Well-defined sharply marginated tumors, ascites, renal masses, hydronephrosis, absence of lymph node disease. Pleural effusion is most common chest abnormality.

Klebsiella – most common cause of Gram-negative pneumonias; community acquired. 5% of adult pneumonias (usually in predisposed). Propensity for posterior upper lobe/ superior lower lobe. Dense lobar consolidation. Bulging fissure – characteristic but unusual. Empyema (common cause). Patchy bronchopneumonia may be present. Uni or multilocular cavities in 50% within 4 days. Pulmonary gangrene is rare. Acute pneumococcal pneumonia may appear similarly.

Mycoplasma – primary atypical pneumonia. Most common cause of non-bacterial pneumonia. Mild course, usually 2-3 wks. Only 10% of infected subjects develop pneumonia. 10-33% of all pneumonias. Autumn peak. Most common in ages 5-20 yrs.
 Mild symptoms of cough, low fever, malaise, otitis
 Mild leukocytosis
 Most common respiratory cause of cold agglutinin production (60%)
 Pulmonary infiltrates show a significant lag time; fine interstitial infiltration from hilum into lower lobe (earliest changes). Alveolar infiltrates: unilateral L>R airspace consolidation in segmental lower lobe in 50%, bilateral in 10-40%.
 Small pleural effusions in 20%
 Hilar adenopathy (rare)

Dahnert, etc.

31. A 40 yr old man was in an MVA and hit the steering wheel with his chest. He has multiple contusions, precordial chest pain, and a holosystolic murmur. What is the most likely cause?
A. aortic laceration
B. papillary muscle rupture
C. aortic valve rupture
D. rupture of the interventricular septum

Answer: A

Classic deceleration injury (as we all are well aware) is traumatic aortic tear. Sxs include interscapular severe chest pain, dyspnea, dysphagia, hypertension of upper extremities, bilateral femoral pulse deficit, systolic murmur in 2nd left parasternal interspace. Papillary muscle rupture and rupture of the interventricular septum are associated with myocardial infarction (see #19).

Dahnert

32. False aneurysms are concerning because:
A. tend to rupture

Chronic posttraumatic aortic pseudoaneurysm = aneurysm existing for > 3 months. 2-5% of patients surviving aortic tear > 24-48 hrs. Symptom free period of months to years. Delayed clinical symptoms. Usually at level of ligamentum arteriosum filling AP window. May develop CHF, partial obstruction of aortic lumen, bacterial endocarditis, aortoesophageal fistula, aortic dissection, obstruction of tracheobronchial tree, systemic emboli. Prognosis = enlargement and eventual rupture. 10 yr survival is 85% with surgical repair and 66% without repair.

Dahnert

33. A 34 yr old asymptomatic construction worker has a routine cxr, which reveals unilateral pleural effusion, which resolves on its own in 6 weeks. Six months later, the patient presents with a focal upper lobe lung opacity. What is the most likely cause?
A. lung cancer
B. TB
C. PE
D. Viral infection
E. Asbestosis

Answer: B

In primary TB, unilateral pleural effusions may be seen in 25% of cases. This is usually associated with parenchymal disease, however. Pleural effusions may complicate pulmonary infection or occur as the primary manifestation of disease. Postprimary TB is due to reactivation and tends to occur in the apical and posterior segments of upper lobes and superior segment of lower lobe. Reactivation usually occurs years after the primary disease in the setting of weakened host defenses from aging, alcoholism, diabetes, cancer, or HIV. Of the choices given, TB is the only disease likely to present in this manner.

Brant & Helms

34. Which sequence is best to study blood flow dynamics on cardiac MR?
A. STIR
B. Gradient recalled echo
C. T1
D. T2
E. Phase contrast

Answer: B

Dr. Ed (Who’ yo MRI Daddy) Jackson says this is a poorly worded question. Nonetheless, he would go with B.
“Gradient-echo or fast-field echo images impart bright signal to coherently flowing blood, creating a “white blood” appearance similar to contrast studies. With gradient recalled echo technique applied, motion studies can show flowing blood as well as myocardial contractility.”

Brant & Helms


2000
35.Regarding pts with AIDS with TB what is false
a. miliary pattern is common
b. hilar/ mediatsinal nodes are seen with rim enhancement
c. may present with adenopathy alone
d. respond to antituberculous meds with same efficacy as immunocompetent pts.

Ans. D. these patients have more drug resistance

36.Azygous vein enlargement is maximal with:
a. Supine position with valsalva maneuver
b. Supine position with mueller maneuver
c. Upright position with valsalva maneuver
d. Upright position with mueller maneuver

Ans: B. Mueller maneuver involves forced inspiration against a closed glottis thus is like reverse valsalva. It increases venous return as does supine position.

37. With regard to a cardiac transplant (orthotopic), which of the following native structures are left in place:

a. left atrium
b. coronary sinus
c. pulmonary valve

Ans: Back wall of LA, and also a part of the RA as a biatrial anastamosis is performed

37. Concerning cardiac surgery, which of the following are true:
a. mitral annuloplasty reverses the effects of calcific mitral stenosis.
b. Coronary ostial markers are not necessary when the coronary ostia are occluded
c. In repairing VSD’s, the patches are on the low pressure RV side
d. The Jantene or switch procedure for transposition involves switching of the venous inflow to the atria

Ans: C: TRUE
A: false: used for M. Regurgitation. D: False: Mustard and Senning are the atrial switch operations while Jantene is arterial switch operation.

39. Question concerning endocardial cushion defects not well remembered

However, page 524 Dahnert.
Associated with Down 25%,Associated with Asplenia/polyspenia
Associated with meso and dextrocardia
- Partial: ASD ostium primum, cleft Mitral Valve, short chordae. Right AV Valve normal.
- Intermediate
- Complete: ostium primum ASD and post VSD and RV/LV one common valve
Chest Xray: Shunt vascularity large PA and heart

40. Which of the following increase lung compliance:
a. CHF
b. Alpha1 Antitrypsin Def.
c. Pulm. Embolism
d. Scleroderma

Ans: B. edema, fibrosis and granulomas all decrease compliance (i.e. Increase stiffness) while emphysema increases compliance.

Alpa1 antitrypsin def. Is a autosomal recessive disorder with panacinar emphysema predominantly bibasilar.

41. A patient with dyspnea presents with a chest Xray showing parallel hilar vessels in the left upper lobe. Next step in management is:
a. Bronchoscopy
b. Biopsy
c. Tube thoracostomy
d. Video assissted thoracostomy

Ans: A. such Xray may mean parallel tram lines or bronchi. Possibilities are upper lobe bronchiectasis (carcinoid, ABPA{CENTRAL}, Cystic fibrosis). High res. CT and then Bronchoscopy may be done.
also non asthamatic patient with atelectasis should be first suspected of having endo bronchial lesion thus needing bronchoscopy.

42. The pulmonary acinus is located distal to:
a. the terminal bronchiole
b. the respiratory bronchiole

ANS: A. The acinus includes 3 or more generations of respiratory bronchioles, alveolar ducts/sacs and alveoli.

43. Apatient develops gangrene of the right middle lobe after right upper lobectomy. This is most likely secondary to:

a. torsion
b. Pulm. Embolism
c. Interruption of superior pulm vein
d. Interruption of interlobar pulm artery

ANS; A. pulm gangrene results from arteritis, vascular thrombosis and/or vascular compression.

44. Malignant Mesothelioma grossly and histologically is most similar to:
a. TB
b. Uremia

Ans. Differentiation from metastatic adenocarcinoma is virtually impossible.

45. What is the age at which we have a full complement of alveoli:
a. 32wks
b. 2yrs
c. 8yrs

ANS: B. development begins at 28 days post conception .
Birth to 15 yrs diameter of airways increases, cartilage and muscles form. Lung parenchyma growth is principally due to new units in infancy and to increase in size in rest of child hood.

46. What do cyclosporine and dilantin toxicity have in common:
a.cardiogenic pulmonary edema
b. non cardiogenic pulmonary edema
c.others

ANS: C. both cause hilar and mediastinal lymphadenopathy along with methotrexate. Both also supress normal immune suveillance resulting in unbridled lymphoproliferation either benign or malignant (hodgkins or NHL)

47. Which disease causes valvular heart disease:
a. Hepatic carcinoid
b. Others

ANS: A. carcinoid syndrome requires the liver metabolism to be bypassed with liver mets or with primary pulmonary or ovarian carcinoids. Causes right heart endocardial fibroelastosis and TRegurg, PV stenosis and RHF.

48. Question not well remembered, about coronary artery calcifications:
a. seen in most people over 70
b. seen in soft plaques

ANS: xray detection rate 42%. calcification in asymptomatic patients indicates more than 50% stenosis and in symptomatic people it indicates more than 75% stenosis. It occursin the hemorrhagic areas of atheromatous plaques.

49. Which is not an agent for pleurodesis:
a. talc
b. alcohol
c. bleomycin
d. tetracycline
e. doxycycline

ANS: B

50. Concerning Left Atrial myxomas, which of the following are true:
a. can mimic AV disease
b. when it arises from the septum it is usually not mobile
c. usually arises from the atrial wall
d. has decreased echogenecity on ultrasound

ANS: All False. Mimics mitral valve disease. Attaches to the interatrial septum most commonly via a stalk. Is hyperechoic.

51. Question about how to tell difference between heart dilatation (CHF) from pericardial effusion.

Look at pulm vessels: If no redistribution/ IPE or APE initially then Pericardial effusion.



Questions 52-68

52. Which of the following is most likely to be associated with development of cancer?
A. Barrett’s esophagus.
B. Granulomatous colitis
C. Gardener’s syndrome.

Answer: C (patients with Gardener’s have almost 100% risk of developing colon cancer. Barrett’s has 10% and GC or Crohn’s has similar to UC +/- 10% per year after 10 years of having the disease. 20% of UC patients will have multicentric colon tumors).
GI Radiology, Eisenberg, 721.

53. A patient with a non-dominant right coronary artery undergoes a bypass for an occlusion of the RCA. 5 years later she presents with chest pain. ST depression of 2-3mm, and a large, posterior wall defect on cardiac imaging. The most likely etiology for this finding is:
a. Graft occlusion with ischemia
b. Graft occlusion with infarct
c. Graft stenosis with ischemia distal to the graft
d. New left coronary disease
e. Graft stenosis with infarct.

Answer: D
Dominance refers to the artery that supplies the diaphragmatic aspect of the interventricular septum and the left ventricle (posterior aspect). 85% of the population have a larger RCA than LCA and the RCA gives rise to the AV node artery. In 10% the LCA is larger and gives rise to this artery. In 5% they are codominant (Primer, 105). The posterior descending artery is a branch of the RCA normally, but may be a branch of the LCx if the left is dominant (Netter, 205).

Cardiac vascular distribution is:
LAD- septum, anterior wall and apex
LCx- lateral, posterior and posterior inferior walls and apex
RCA- inferior and posterior inferior walls and the right ventricular wall.
Left main coronary artery- anterior and posterolateral walls and septum.
So, in non-RCA dominant system, the Left system supplies much of the posterior wall, particularly the LCx.
Requisites, p79

54. Indium-111 is preferred over Ga-67 in which of the following scenarios?
A. Disc space infection
b. Splenic abscess
c. Cellulitis
d. Active inflammatory bowel disease
e. Opportunistic lung infection

Answer: B, C, D
The general rule is that In-111 is used for acute infections because it labels WBC which are active in acute processes. Ga-67 is used in the chronic state. Ga binds to transferrin in the blood and it is delivered via increased vascular permeability (like in abnormal vessels in tumor or leaky vessels in infection/inflammation. Although bacterial uptake and binding to WBC occur, it is not the major mechanism of localization. For intraabdominal processes, In-111 is preferred because Ga is excreted in the bowel and can interfere with the interpretation. This includes inflammatory bowel disease, abscess and post surgical patients. Since the spleen normally takes up Ga, In-111 is used for splenic abscesses. Now, spine infections tend to be a more chronic process and the labeled WBC have poorer sensitivity than Ga, with false negatives of 10-40%, so Ga is preferred. For lung infections and imaging in HIV, Ga is used because In-111 is very nonspecific. (Requisites, 188-190)

55. Which does not take up Octreotide?
A. Pheochromocytoma
b. Meningioma
c. Renal cell carcinoma
d. Medullary thyroid cancer
e. Islet cell tumor of the pancreas

Answer: C
Octreotide is a somatostatin analog that has FDA approval to be used in the treatment of symptoms of metastatic carcinoid and vasoactive intestinal peptide tumors and to suppress growth hormone in acromegaly.
In-111 pentetreotide (octreoscan) is used for imaging of neuroendocrine tumors. It images APUDomas including pituitary tumors, pancreatic islet cell tumors, carcinoids, medullary thyroid carcinoma, paragangliomas, pheos and neuroblastomas. Other tumors with somatostatin receptors can be imaged: meningiomas, astrocytomas, malignant thymoma, breast cancer, SCLC and lymphoma. Requisites, 224.

56. A 20-year old patient presents with an elevated T4, thyroid uptake of I-131 of less than 1%, sore throat, and preceding viral illness. The most likely diagnosis is:
a. Graves disease
b. Hashimoto’s thyroiditis
c. Acute suppurative thyroiditis
d. Subacute non-suppurative thyroiditis

A. F (will have increased Iodine uptake)
B. F (May have increased or decreased uptake, but usually asymptomatic)
C. F (Rare, bacterial thyroiditis. Thyroid is typically enlarged and tender)
D. T (This is the classic presentation of subacute thyroiditis also called granulomatous
or de Quervain’s. The etiology is not definite, but speculated to be viral since patients may give a history of viral type symptoms and usually present with neck tenderness or sore throat. The uptake is decreased or absent in the affected part of the gland).
Requisites, 376.

57. In a Meckel’s scan, what drug is given to block excretion from the gastric mucosa?
A. Pentagastrin
b. Cimetidine

a. F ( this is used to help trap the Tc at the ectopic mucosa by making it more active).
B. T (this is used to block excretion, it is an H2 blocker).
Requisites, 243.

58. With regards to HIDA scans, which of the following are true?
A. HIDA is conjugated by hepatocytes
b. Bowel activity distinguishes neonatal hepatitis from biliary atresia.
C. HIDA is taken up by adenoma.

A. False B. True C. True
Tc-99m IDA has the same hepatocyte uptake, transport and excretion pathways as bilirubin. The IDA compounds are stable in vivo, and in contrast to bilirubin, are excreted in their original radiochemical form WITHOUT being conjugated or undergoing significant metabolism. Since they are excreted into the biliary system, it is used to diagnosed biliary atresia in the newborn, in a positive study there will be no excretion and no visualization of the bowel (Requisites, 245). Now, Dr. Lamki and Dr. Barron both said that the Requisites is wrong in terms of the adenoma, that because the adenoma contains hepatocytes, it should have uptake in the HIDA scan. The same holds true for FNH, only that FNH also has the Kupffer cells so will be hot or normal on sulfur-colloid and the adenoma will be cold. (Wilson, 67).


59. Concerning Gamma cameras, which of the following is true?
A. Extrinsic field uniformity is measured with Cobalt 57 in a flood source with the collimator on.
B. Basically know how intrinsic and extrinsic field uniformity is measured.

Answer: A is True. Cobalt 57 is used for extrinsic field uniformity which by definition means with the collimator on. For the intrinsic, which means without the collimator, a point source of 200-400 uCi Tc-99m is used. Some places use a phantom filled with a uniform solution of Tc-99m for the extrinsic test as well. Req, 28.

60. PET scans- not remembered well. Which is true?
A. On-site cyclotron needed
b. PET uses a different isotope that F-18 glucose

Answer: A. True B. I don’t understand the sentence, but PET uses F-18 FDG(fluorodeoxyglucose) among others like ammonia and C11 and oxygen.

61. Alzheimer’s disease on HMPAO demonstrates:
a. Decreased uptake in the parietal lobes
b. Decreased uptake in the frontal lobes
c. Increased uptake in the parietal lobes
d. Increased uptake in the frontal lobes.
Answer: A. Alz’s has decreased uptake in the temporoparietal distribution. Pick’s dementia/disease will have decreased uptake in the frontal lobes. Multi infarct dementia and AIDS encephalopathy look similar with multiple areas of decreased activity. Req., 311.

62. All of the following may lead to a V/Q mismatch with perfusion decreased more than ventilation except:
a. Radiation therapy
b. Hilar adenopathy
c. Mediastinal fibrosis
d. Interstitial pneumonitis
e. Hypoplastic left pulmonary artery

Answer: D
Conditions associated with V/Q mismatch include: acute/chronic pulmonary embolism, other causes of embolism (drug abuse, iatrogenic), bronchogenic carcinoma (other tumors), mediastinal or hilar adenopathy with obstruction of pulm art or veins, hypoplasia or aplasia of pulmonary artery, Swyer-James syndrome (some cases), post radiation therapy and vasculitis. Interstitial pneumonitis is not included.

Another table lists the causes of MATCHED V/Q defects:
COPD
bronchitis and bronchiectasis
blebs and bullae
Congestive heart failure
pulmonary edema
pleural effusion
asthma
pulmonary trauma, hematoma
inhalation injury (may show as interstitial pneumonitis)
mucus plug
bronchogenic carcinoma
Req., table p.159 and 160.

63. Question about MUGA, not remembered.

This will be MUGA in a nut shell from Requisites, page 93 +:

Radionuclide ventriculography is done to evaluate global and regional ventricular function. The pharmaceutical of choice for equilibrium gated blood pool imaging is Tc99m-labeled RBC. Labeling can be done using three approaches: in vivo (lowest labeling, 60%, and sensitivity), modified in vivo (intermediate labeling, 90%, and sensitivity) and in vitro (best, >95%, the one they use at LBJ and HH). Another agent is Tc99m-HSA of human serum albumin. This has the advantage that can be prepared ahead of time, but is leaks out of the blood quickly, so imaging can not be repeated like with the RBC that they recirculate over and over and multiple images can be acquired. For imaging they do a first pass study with or without EKG gating and they obtain 16-30 frames/sec for about 30 secs. If the patient has a cardiac condition, they image for 60 secs. The patient is typically positioned in the RAO view with 20-30 degrees of angulation. This can measure EF from the RV and LV.

The other approach is the Equilibrium gated blood pool study. This is preferred because it has better counts and more images that are added. The EKG is synchronized to the R wave. The cardiac cycle is divided into 16-24 frames and the duration of each frame is 40-50 msec. They image about 100-300 cardiac cycles, having about 250,000 counts/frame. For studies at rest (those done for adriamycin toxicity for example) multiple views are used including, shallow 10 RAO, a 30-60 LAO and LPO. The LAO angle is the one that best divides the RV and LV activities. For stress imaging, the camera is left in the LAO position.

64. Compared to Tc-99m labeled RBCs, Tc99 sulfur colloid is:
a. Better for UGI bleeds
b. Quicker in preparation
c. Can re-image in 24 hrs
d. Better for intermittent hemorrhage
e. More sensitive for colonic flexure bleeds

Answer: B (SC comes prepared in a kit, the RBC need to be labeled).

All the other alternatives are true for RBCs. SC is taken up by the liver and spleen, so not sensitive for UGI bleeds and for the same reason colonic flexure bleeds. SC needs to be imaged within 10 mins of injection because it disappears quickly, so its not good for intermittent bleeds or to reimage later (Dr. Lamki calls it the nuclear medicine angiogram). Now, SC still can detect smaller bleeds, 0.05-0.1 ml/min vs 0.1-0.35 ml/min for RBCs. Verbatum: “There is however, one clinical situation in which SC is preferred: patients with collateral abdominal vessels that can obscure bleeding sites in labeled RBCs scans. In these patients, the sequestration of the SC in the liver allows the identification of a focal intraluminal collection of tracer otherwise obscured by the plethora of collateral vessels” (Wilson, 69).

65. Regarding gastric emptying:
a. Solids are more sensitive to gastroparesis than liquids
b. Have to average the anterior and posterior images
c. Liquids display linear emptying

Answer: A. True B. False C. False
In gastric emptying studies, food moves from the fundus, which serves as a reservoir, into the antrum where the food is grinded. Subsequently, it is emptied into the duodenum. Lag phase is the time it takes for the antral grinding before it moves into the duodenum. The solid study is more sensitive. The liquid study is always normal when the solid one is normal. A liquid only study should be reserved for patients who cannot tolerate solids. In terms of how they image there is no set protocol. The gold standard is to use 2 cameras, obtain ant and post views and get a “geometric mean”. Other places use an LAO view and others just the ANT approach (Wilson, 77 and Requisites,277). The point is to do every study the same way. So, I guess B is False since you don’t HAVE to average the two (in LBJ and HH they do ant approach). The emptying curve is linear for solids and exponential for liquids.
66. The least likely cause of a superscan:
a. Hyperparathyroidism
b. Metastatic breast cancer
c. Chronic renal failure
d. Other choices

Answer: Causes of superscan include:
renal osteodystrophy
mets (breast, prostate)
lymphoma
hyperparathyroidism
hyperthyroidism
osteomalacia
marrow expanding processes (anemia, sickle cell, myelofibrosis, neoplastic infiltration)
Paget’s
OSTEOPOROSIS has no role in bone scans or superscan for that matter, so I imagine this may have been one of the “other choices”.

67. Bone scan least likely to show uptake in the liver
Don’t remember the choices, but osteosarc/colon mets was not a choice.

Answer: Purely based on old questions these have been alternatives throughout the years:

contamination of the agent with aluminum
mets from breast, colon or osteosarc
contamination of the mixing vial by air
focal nodular hyperplasia
hepatitis

From these choices FNH or hepatitis should not show uptake in the liver. There may be abnormal liver uptake in amyloidosis or hepatic necrosis (Req., 142). Al and air cause clumping and focal areas of increased activity.


68. Question on non-Q wave MI

This may have been a question of diagnosis of AMI even without EKG changes. Info is from Requisites, 88 +: The major advantage of myocardial perfusion imaging with Tl201, Tc99m sestamibi, and Tc99m tetrofosmin (used more now because is cheaper than mibi) over Tc99m pyrophosphate is that the study is positive immediately post infarction. With the pyrophosphate, the study is more sensitive >72 hrs after the MI, it is basically insensitive <24hrs or >10 days. The sensitivity with the other agents is >90% for transmural infarctions, after 24 hrs it decreases to +/- 60%. Also it is slightly less sensitive for nontransmural or non Q wave infarcts (Guiberteau, 97).


Mammo

69. After core needle biopsy of a speculated breast lesion, pathology reports a radial scar. The next step is:

A. excisional biopsy
B. follow-up mammogram in six months
C. repeat core biopsy
D. routine follow up

Answer: A
The appearance of a radial scar can be indistinguishable from malignancy. Radial scars have dense radiating lines that are usually very long greater than or equal to 1cm, without a central mass. The radiating lines are interspersed with trapped fat, forming lucent zones near the center of the area. Radial scars should be biopsied to make a secure diagnosis. Needle biopsy has been found to be an unreliable method of diagnosing these lesions. Excisional biopsy is needed to to make the definitive diagnosis. Kopan’s page 380,299.


70. Which cancer most often arises in a cyst?

A. Medullary
B. Mucinous
C. Papillary
D. tubular

Answer: C
Carcinoma is rarely found in a cyst, however, when they occur, most intracystic cancers are papillary. When cyst fluid is maroon or brown, previous hemorrhage should be suspected. Hemorrhage into a cyst may be idiopathic, secondary to an intracystic papilloma, or due to a rare intracystic cancer. Cysts containing old hemorrhagic fluid should be excised because of the small possibility of intracystic carcinoma. The majority of focal intracystic lesions, prove to be benign papillomas. (Kopan’s pages 538-540)

71. Which portion of the breast is least visualized in a standard MLO image?

A. upper inner
B. upper outer
C. lower inner
D. lower outer
E. axillary tail

Answer: A

72. Which of the following breast cancers is the most common cause of a well-circumscribed, malignant mass?

A. medullary
B. tubular
C. colloid
D. invasive ductal carcinoma
E. mucinous

Answer: D

The most common cancer that grows as a circumscribed mass is infiltrating ductal carcinoma NOS. Lymphoma, colloid carcinoma, and papillary carcinoma can on occasion appear as round, smoothly marginated lesions. Papillary cancers are frequently intracystic, and the cyst accounts for the smoothness of the margin. (Kopan’s page 279)

73. The appearance of the skin in a patient with inflammatory breast carcinoma is due to which of the following?

A. inflammatory cells within the breast.
B. Obstruction of dermal lymphatics by tumor cells
C. Local invasion of the dermis

Answer: B

Thickened skin invariably accompanies inflammatory carcinoma, a highly aggressive manifestation of breast cancer with poor prognosis, in which tumor is found permeating the dermal lymphatics. By itself, however, the thickened skin is a nonspecific finding, and the diagnosis of inflammatory carcinoma is based on the clinical findings with erythema and increased skin temperature over 1/3 of the breast surface. Get peau d’orange appearance. Radiographically, have nonspecific skin thickening . The overall trabecular pattern of the breast appears thickened due to congestion in the intramammary lymphatics. On occasion, the underlying tumor mass that is not detectable on clinical examination, is visible mammographically. (Kopan’s page 341)

74. Core biopsy of a breast lesion demonstrates atypical hyperplasia pathologically. The next step should be:

A. re-biopsy
B. excisional biopsy
C. follow-up mammo in 6 months
D. follow-up mammo in 1 year.

Answer: B

Several studies have demonstrated that is the histology of a core needle biopsy is read as atypical hyperplasia (a high risk, but benign lesion), excisional biopsy is warranted because may prove to be breast cancer at excision. (Kopan’s page 226)

75. Mammographic findings after reduction mammoplasty include all of the following except:

A. skin thickening
B. fat necrosis
C. shift of glandular tissue superiorly
D. nipple moved to a higher position

Answer: C

The most common way of reduction mammoplasty, involves a long incision along the inframammary fold and incisions extending from it to the 6 o’clock edge of the areola and then around the areola. Tissue is removed from the bottom of the breast and between the vertical incisions. The nipple areolar complex is moved up into a keyhole extension of the vertical incisions, which are then brought together in the midline to reform the smaller breast. The fibroglandular tissues can appear redistributed from the usual upper breast location to the newly formed lower breast. There is often a swirling configuration to the remolded tissues, and isolated islands of breast tissue can be seen. Fat necrosis is relatively common with the extensive surgery involved, and oil cysts, as well as benign calcifications can be seen. (Kopan’s pages 460-462)

76. The most characteristic feature of a lymph node on US:

A. well circumscribed
B. oval
C. ill defined margins
D. notching
E. lobulated mass
Answer: D

Normal lymph nodes are reniform or oval, and have an echogenic fatty hilus. In depicting the fatty hilus within a lymph node, ultrasound is not reliable in excluding malignancy in a woman with breast cancer. Normal sized lymph nodes that are infiltrated with tumor may appear identical to normal nodes. A hypoechoic cortical focus or an irregular cortical margin may be found in a lymph node, suggesting a metastatic deposit. Normal intamammary lymph nodes are in the upper outer quadrant, and have a hilar notch. (Cardenosa, p 135, Rumak pages 759-760)

77. All of the following occur with in the ducts except

A. invasive ductal ca
B. adenosis
Answer: B

The most common benign lesions such as fibroadenomas, cysts, and adenosis, which arise in the lobule. The lobule comprises the multiple terminal branches of the duct network that are blind-ending and form glandular acini that resemble the fingers of a glove. Adenosis is a benign proliferation of the stromal and epithelial elements of the lobule producing an increased number of acinar structures. Adenosis is a benign process, but when associated with fibrous architectural distortion (sclerosing adenosis), it can be confused with malignant change.

Lesions that develop in the major ducts of the breast include Paget’s disease, duct ectasia, solitary large duct papillomas. Atypical ductal hyperplasia arises at the level of the terminal duct lobular unit (TDLU). Intraductal carcinoma (DCIS), and invasive or infiltrating ductal carcinoma arise at the level of the TDLU. (Kopan’s pages 513-516)


78. Screening mammo showed a well-circumscribed density only on the CC view. Ultrasound was negative. The next step should be:

A. biopsy
B. lateral
C. nothing, return to routine screening

The answer to this question is to first confirm that the lesion is real with a spot compression view, or a roll cc view. If the lesion is real, circumscribed masses with well defined borders are an uncommon sign of malignancy; only 2% of solitary masses with smooth margins are malignant. Therefore, it falls into a Birads 3 with a six month follow up.

79. A 55-year-old woman is S/P MRM. On her f/u mammo there are 7 areas of tea cup calcifications. Most likely diagnosis is:

A. fat necrosis
B. recurrence
C. not remembered completely

Answer: Milk of calcium calcifications

Milk of calcium calicifications are consistent with sedimented calcifications in cysts. On the CC image they are often less evident and appear fuzzy, round, amorphous deposits while on the horizontal beam lateral image, they are sharply defined, semilunar, crescent shaped, curvilinear (concave up) defining the dependent portion of cysts.

Additional benign calcifications include skin calcifications which have a polyhedral shape with lucent centers. Vascular calcifications which have parallel tracks or linear tubular calcifications. Course or popcorn-like calcifications produced by an involuting fibroadenoma. Large rod-like calcifications which are > 1mm continuous rods, that may branch. Represent benign secretory disease (plasma cell mastitis) or duct ectasia. Spherical or lucent centered calcifications. Range in size from <1mm to >1cm. Rim or eggshell calcifications are very thin benign calcifications that appear as calcium deposited on the surface of a sphere—fat necrosis and cysts. Suture calcifications, and dystrophic calcifications. Dystrophic calcifications are irregular, >0.5mm, and often with lucent centers. Associated with irradiated breast, or in the breast following trauma. (Kopan’s pages 773-776)

80. A 40-year-old woman has a nonpalpable well-circumscribed mass seen in her right breast on her first screening mammogram. She is asymptomatic. Ultrasound in this area is unable to demonstrate this mass or any other abnormality. Which one of the following should be done next?

A. incisional biopsy
B. core biopsy
C. 6 month follow up
D. mastectomy

Answer: C

The probability of malignancy for round or oval masses with well circumscribed margins is 2%. There should be no evidence of infiltration on magnification mammography. Most agree that it is reasonable to follow them at a short interval if they are found on a first mammogram and intervene only if they enlarge or demonstrate any other significant changes. (Kopan’s pages 280-281).

81. The goal of a study evaluating screening mammography is to evaluate:

A. increased survival rate
B. mortality reduction
C. interval carcinoma
D. the increase in early detection of cancers
E. the increase in alter detection of cancers

Answer: B

Early detection remains the only proven method for reducing the mortality from breast cancer, and the mammographic screening of asymptomatic women remains the only efficacious method for detecting earlier stage, clinically occult breast cancer. (Kopan’s pages 52-53)

82. To decrease motion unsharpness in mammography:

A. increase the phototimer density
B. decrease the KVP
C. other choices not well remembered

To decrease motion unsharpness, use compression to immobilize the breast, and decrease exposure time. If kVp is increased, the mAs is decreased, and you decrease exposure time, which decreases motion unsharpness. Geometric unsharpness is reduced by moving the focal spot as far away from the object and detector as possible, reducing the size of the focal spot, and compression of the breast (moving the breast as close to the detector as possible). Kopan’s pages 140-141

83. All of the following are advantages of core biopsy as compared to needle biopsy except:

A. the biopsy can be sonographically guided
B. the pathologist has an easier time analyzing the specimen/is better at reading core specimens
C. more frequently gives benign diagnoses
Answer: C

Core biopsy provides enough tissue that allows histologic information, so that it can be interpreted by most pathologist. Invasive cancers can be distinguished from in situ lesions. Core needle biopsies are more practical than cytology from fine needle aspiration biopsy for reducing the number of open surgical biopsies that will be needed. A benign finding may be confidently diagnosed with core biopsy. (Kopan’s page 708)

84. Well-defined mass of mammo. No Calcifications and was not present one year ago. What is the percentage chance of malignancy?

A. 1-2 %
B. 5%
C. 10%
D. 15%

answer: A (see question #80)

85. An excisional biopsy was performed for calcifications. Specimen radiograph contained two clusters of calcifications. The pathologist reported no malignancy and no calcifications. What is the next step?

A. repeat excisional biopsy
B. mammography follow up in 1 year
C. radiograph the paraffin block
D. immediate post-operative mammography

Answer: C.

The location of the calcifications is important because that is the section of specimen that you want the pathologist to carefully search for malignancy.








103. What is the most common tumor to seed the subarachnoid space.

A. medulloblastoma
B. ependymoma
C. astrocytoma
D. hemangioma

Medulloblastomas are the second most common pediatric brain tumor (second only to astrocytomas) and the most common pediatric posterior fossa tumor. Peak occurrences are at 4-8 years and at 15 to 35 years, but 75% of patients are less than 15 years of age. They are found only in the posterior fossa. They are usually solid, hyperdense masses on CT. 20% calcify and hemorrhage is rare. Easily differentiated on noncontrast CT from astrocytoma because the astrocytoma will usually be hypodense whereas the medulloblastoma will almost never be hypodense. Medulloblastomas are challenging therapeutically because of their tendency to spread via the subarachnoid spaces which has occurred up to 50% of the time at the time of diagnosis (Brandt & Helms:pg 131)

Most ependymomas are benign and subarachnoid seeding is rare. In fact, its presence should suggest the possibility of a malignant ependymoma.

Astrocytoma comprises a group of neoplasms from low-grade to high-grade with GBM being the most malignant. And of GBM, only approximately 2% metastasize by CSF spread. (Dahert 4th edition: pg 234)

Hemangiomas are typically seen in the vertebral bodies. Assuming that this question was referring to hemangioblastomas, these are vascular lesions that may have both a cystic and solid component, with the solid component enhancing intensely. On MRI flow voids can be identified at times within the tumor and prominent posterior draining veins can also be seen. There is a strong association with Von Hippel-Lindau disease.( Requisites: pg 487) I could not find any significant tendency toward CSF spread.

Answer A: Medulloblastoma.


104. Concerning cranial nerve 7:
A. exits via the stylomastoid foramen
B. carries gustatory to the posterior 1/3 of the tongue
C. commonly enhances on Gd-MR
D. other choices

The facial nerve motor nuclei reside deep in the reticular formation of the caudal part of the pons. It innervates the stapedius, stylohyoid, posterior belly of the digastric, occipitalis, buccinator, muscles of facial expression and platysma.

The sensory nucleus provides sensation to the anterior 2/3 of the tongue via the chorda tympani.
The nerve itself courses from the lateral aspect of the pontomedullary junction anterolaterally in the cerevellopontine angle cistern to to the internal auditory canal to synapse on the geniculate ganglion giving off the greater superficial petrosal nerve which innervates salivation. From there it turns posteriorly traveling below the lateral semicircular canal then inferiorly to exit through the stylomastoid foramen. At different points along its course it gives off the motor and sensory innervations noted above.(Dahnert: pg 206, Requisites: pg 339-341)

Could not find information on the specific appearance of CN VII on Gd-MR.

Answer: A, exits via the stylomastoid foramen.


105. A young woman presents with sudden loss of sight. MRI shows a cystic sellar lesion with sedimentation level that compresses the optic chiasm. The most likely diagnosis is:

A. craniopharyngioma
B. pituitary adenoma
C. pituitary apoplexy

The key to answering this question is recognizing the sudden nature with which loss of sight occurred and also the presence of the sedimentation level on cross-sectional imaging.

Certainly all three of these are capable of exerting mass effect on the optic chiasm and producing a bitemporal hemianopsia. However, in the cases of craniopharyngioma and pituitary adenoma, vision loss would probably be a more slowly developing process.

Pituitary apoplexy is a syndrome that appears suddenly with combinations of ophthalmoplegia, headache, visual loss, and/or vomiting. This is usually caused by massive hemorrhage into a pituitary adenoma as is seen in patients who are on bromocriptine for pituitary adenoma.( Bromocriptine increases the likelihood of hemorrhage.) But can also be caused by pituitary infarction as seen in Sheehan syndrome which is post-partum infarction/necrosis of the anterior pituitary gland.

With hemorrhage into the pituitary, a sedimentation level or fluid-fluid level can be seen on MRI due to blood products at different stages of evolution. (Neuro requisites and Dahnert)

Anwer: C, pituitary apoplexy.


106. Contra-indications to MRI:

A. S/P CABG within the last 7 days
B. Metallic foreign body in the eye
C. Hip prosthesis

Given the number of orbit CT’s that we’ve all read in the ER prior to performing MRI’s, I think the single best answer here is clear. Hip prostheses are not a contraindication to MRI. At MD Anderson, however, we do wait 6 weeks post hip replacement. As for the CABG question, I’m assuming the concern would be the sternal wires, but I could not find any information on this.

Answer: B, metallic foreign body in the eye.



107. 2nd question on contra-indication to MRI

D. coronary ostial marker
E. prosthetic mitral valve
F. cochlear implant

Cochlear implants are considered unsafe for insertion into the magnetic field and are a contraindicated device. None of the prosthetic heart valves currently on the market are considered unsafe including Star-Edwards and St. Judes models which do have weak associated ferromagnetic fields (MRIsafety.com) Could not find any information on coronary markers.



108. Wernicke’s encephalopathy is associated with abnormalities in

a. hippocampus
b. mamillary bodies
c. atrophy of the caudate head
d. cerebellar atrophy

Wernicke’s encephalopathy (ophthamoplegia, ataxia, and confusion) is thought to be due to thiamine deficiency and is usually caused by chronic alcoholism. Neuro requisites states that “in addition to generalized cortical and cerebellar vermian atrophy”, other radiographic manifestations include the presence of high signal intensity in the periaqueductal gray matter of the midbrain and third ventricle as well as thalamic lesions which may be associated with mamillary body atrophy. However, most texts refer to cerebellar changes as being directly attributable to alcoholism and not to the encephalopathy. But all texts studied, including AFIP notes make direct reference to mamillary body abnormalities with Wernicke’s encephalopathy.

Also remember that once severe amnesia accompanies Wernicke’s encephalopathy, it becomes Wernicke-Korsakoff’s. (Requisites: pg227, Brandt & Helms: pg 185-187, AFIP notes: Dr. Koeller on acquired white matter dz.)

Answer: B, mamillary bodies


109. A lesion which produces homonymous hemianopsia is located where?


A. cerebrum
B. optic nerve
C. lateral geniculate
D. occipital lobe
E. anterior chiasm

Assuming that this question was remembered correctly, the answer would probably be C, lateral geniculate nucleus.

The visual pathway, as you know, begins in the retina with rods and cones which transmit signal to sensory ganglia that converge to form the two optic nerves. The optic nerves travel posteromedially through the optic canal to the optic chiasm where decussation occurs. The fibers from each nasal retina (aka temporal visual field) cross to join the fibers from the contralateral temporal retina (aka nasal visual field). From there, fibers travel through the optic tracts to their respective lateral geniculate nuclei and then on through the optic radiations to occiptital cortex.

So in theory at least, any unilateral insult in the visual pathway at or posterior to the optic chiasm could affect vision in such a way as to create a homonomous hemianopsia (affecting the same side of the visual field bilaterally). But in reality, this is not so.

A lesion in the anterior midline chiasm would most likey cause a bitemporal hemianopsia as this is where fibers from each temporal field decussate. But this is the right side of one field and the left side of the other.

Obviously, a lesion in the optic nerve itself would only affect one eye.

And it is known that while occipital cortical strokes do affect vision, they typically spare macular vision (vision at the center of the visual field). This is thought to either be secondary to a dual arterial supply to cortical areas of macular vision or some minor decussation of macular fibers.

Answer: C, lateral geniculate.


110. Which is true regarding carotid fibromuscular dysplasia?

A. Usually unilateral
B. Associated with multiple, intracranial saccular aneurysms
C. Involves the internal carotid more so than the external carotid

Fibromuscular dysplasia is described as nonatheromatous fibrous and muscular thickening alternating with dilatation of the arterial wall, which produces an appearance characterized as a “string of beads.” The etiology of the disease is unknown. The most common CNS vessel affected is the internal carotid artery, approximately 2cm or more from the bifurcation. The most common peripheral vessel affected is the renal artery. Multiple vessel involvement is common (bilateral carotid involvement occurs in 60%). Intracranial FMD is rare. However, there appears to be an association between intracranial aneurysms and carotid FMD (approximately 25% of carotid FMD have associated intracranial aneurysms.) (Brant & Helms: pg 640, Requisites: pg 116)

Could not find information on exact type of aneurysms.

Answer: C, and probably B


Gastrointestinal Radiology

111. A 70 year old woman has a 5 cm mass in the pancreatic head that contains central calcifications. This finding was present on a previous study five years ago. Most likely dagnosis is:

A. pseudocyst
B. microcystic adenoma
C. pancreatic adenocarcinoma
D. papillary pancreatic carcinoma

Pancreatic pseudocysts are typically seen in association with acute or chronic pancreatitis, and less often with pancreatic cancer or status post trauma. They are walled off fluid collections usually within the pancreas with fibrous capsules. The wall of the cyst may calcify and there may be internal septations but internal calcifications are not a prominent feature. (Dahnert: pg 603)

Microcystic adenoma of the pancreas is a benign lobulated neoplasm composed of innumerable small cysts (1-20 mm) containing proteinaceous fluid separated by thin connective tissue septa. They comprise approximately 50% of all cystic pancreatic neoplasms. 82% of those found are in patients over the age of 60. Masses can be palpable at the time of Dx. There is an association with Von Hippel-Lindau. Amorphous central calcifications are seen in 33%. These lesions are three times more common in females. It is not uncommon for them to be an incidental finding. (AFIP notes: Dr. Beutow, Dahnert: pg. 597)

Solid and papillary neoplasm of the pancreas (aka papillary-cystic neoplasm) is a rare low-grade malignant tumor. It is seen most often in young ( mean age 25) black and east asian females. It is typically a large well-encapsulated mass with considerable hemorrhagic necrosis and cystic degeneration. There is a predilection for the tail of the pancreas. Calcification is seen centrally in approximately 33%. The prognosis is excellent following excision. ( AFIP notes: Dr. Beutow, Dahnert: pg 611)

Pancreatic Adenocarcinoma (aka pancreatic ductal adenocarcinoma) comprises 80-95% of nonendocrine pancreatic neoplasms. It is the 5th leading cause of death in the United States. Etiologic factors include alcohol abuse, smoking, and hereditary pancreatitis. Those with diabetes are 2X more likely to get the disease than the general population. Mean age of onset is 55 with peak age around the seventh decade. Male : Female = 2:1. At presentation 65% of patients have advanced local disease or distant metastases. The CT appearance is that of a mass with a central zone of diminished attenuation. Only approximately 2% calcify. It can be associated with ductal obstruction. Prognosis is poor with less than 1% 5-year survival.

Answer: B, microcystic adenoma.


112. Question regarding Oriental cholangitis

Oriental cholangiohepatitis (aka recurrent pyogenic cholangitis) is characterized by recurrent attacks of fever, chills, abdominal pain and jaundice. Its exact cause is unknown but is associated with parasitic infection. The disease is endemic to South China, Indochina, Taiwan, Japan and Korea. It is the 3rd most common cause of an acute abdomen in Hong Kong after appendicitis and perfed ulcer. Radiographically, it is marked by dilatation of the proximal intrahepatic ducts in 100% of cases. The ducts fill with nonshadowing soft mudilike pigment stones (64%). Focal strictures of the ducts may also be seen. (Dahnert: pg 572)


113. Which of the following is most characteristic of bowel involvement by Crohn’s disease?

A. jejunal involvement is greater than ileal involvement
B. asymmetric mural wall involvement
C. continuous involvement without skip lesions
D. duodenal involvement is common
E. concomitant ileal and colonic involvement occurs in >50%

The terminal ileum is involved either alone or in combination with other bowel loops approximately 95% of the time. (Dahnert : pg 670)

The classic description of Crohn’s bowel is skip areas of asymmetric bowel wall thickening. Ulcerative colitis, on the other hand, is more continuous and circumferential.

Duodenal involvement occurs in 4-10% of cases. (Dahnert) Not very common.

The colon and small bowel are involved approximately 50% of the time, according to Dr, Rohrman of the AFIP. But he does not specify ileal and colonic involvement.

Answer: B, asymmetric mural wall involvement


114. Areae gastricae refers to:

A. an area of accumulation of gastric glands
B. a pathologic area of gastric folds
C. other choices

The term areae gastricae refers to the detailed pattern of the gastric mucosa as demonstrated by double-contrast technique. Normal areae gastricae varies from a fine reticular pattern to a course nodular pattern. The hallmark of normal is the regularity of the pattern in all areas in which it is visualized. (Brant & Helms: pg 725)


115. Benign gastric ulcers on the greater curvature are most commonly associated with:

A. morphine
B. aspirin
C. prednisone

Prolonged steroid therapy is a known cause of benign gastric ulcers. Aspirin is a know cause of gastritis which can also lead to ulcer formation (Dahnert). An ulcer in the absence of gastritis changes is more likely due to steroid therapy than aspirin. Found no correlation between morphine and gastric ulcers.

Answer: C, prednisone.


116. What cell in the stomach secretes acid?

A. parietal cells
B. chief cells
C. argentaffine cells

Parietal cells produce HCl, intrinsic factor, and prostaglandins. Chief cells produce pepsinogen. Argentaffine cells produce glucagons-like substance, somatostatin, vasoactive intestinal peptide, and 5-hydroxytryptamine.

Answer: A, parietal cells.

117. A 40-year-old woman presents with diffuse abdominal pain, diarrhea and symptoms of intussusception. CT showed? SBFT showed?

A. Crohn’s disease
B. Lymphoma
C. Sprue

Without knowing what the imaging studies showed, the answer to this question is reduced to descriptions of imaging findings in these three disease processes.

Crohn’s disease is often seen on CT as asymmetric (full-wall but not necessarily circumferential) thickening of bowel wall with skip lesions which may involve any segment or combination of segments of the the upper or lower gastrointestinal tract. Areas of bowel wall thickening often exceed 1.0 cm. It is also not uncommon to see areas of fat stranding associated with affected loops. “Cobblestone” is a term commonly used to describe the appearance of the thickened bowel especially the mucosal surface as seen on double contrast examinations. The small bowel may also demonstrate areas of lymphoid hyperplasia as well as nodularity, spasm, stenosis and ulceration. Complications that may be identified on imaging include fistulas, abcesses, strictures, toxic colon and carcinoma. (AFIP notes)

In Hodgkin’s lymphoma, visceral organ involvement is almost always secondary to adjacent node involvement. 90% of patients will have nodal enlargement and mesenteric nodal involvement is rare. (AFIP notes)

In non-Hodgkin’s lymphoma, the GI tract is the most common extranodal site, stomach >small bowel >colon. There is usually mesenteric nodal disease. There is circumferential bowel wall thickening and nodular filling defects. The lumen is often dilated. (AFIP notes)

The classic radiographic appearance of nontropical sprue is dilated small bowel with normal or thinned folds predominantly involving the jejunum. Visible folds show an increased amount of separation. Tropical sprue has the same radiographic appearance as nontropical sprue but is confined to India, the Far East, and Puerto Rico. (Brant & Helms)

118. Which is the most common urinary tract complication of IBD?

A. bladder stones
B. ureteral obstruction
C. vesicoenteric fistulas
D. calculi

Genitourinary complications of inflammatory bowel disease are seen more commonly in Crohn’s than with UC. Urolithiasis is the most common genitourinary complication (2-10%). Other GU manifestations include amyloidosis, obstruction, and fistulas. ( AFIP notes, Dr. Rohrmann)


119. A patient has a 1.5 cm colonic polyp. What is the chance/probability that this is malignant?

A. 0%
B. 1%
C. 10%
D. 46%

According to Dr. Peter C. Beutow of the AFIP, assuming that the polyp represents a small adenoma, the likelihood that it represents carcinoma is 10% based on its size and the following chart....

Size probability of CA
<1cm 1%
1-2cm 10%
>2cm 30-50%

Answer: C, 10%.



Cynthia’s questions 120-137:
120. Which of the following statements regarding Helicobacter pylori is false?

A. ulcers may occur in the pylorus and first portion of the duodenum
B. associated with gastric carcinoma
C. associated with gastric lymphoma
D. associated with Zollinger-Ellison syndrome.

Answer: D
Helicobacter pylori are a gram-negative spiral-shaped bacillus. It has a prevalence which increases with age: > 50 % of Americans > 60 years of age. The vast majority of patient’s are asymptomatic. If symptoms are seen, then they can include dyspepsia and epigastric pain. Patients can have gastritis with thickened gastric folds, enlarged areae gastricae, and polypoid gastritis mimicking malignant tumor.
- gastric ulcer (60 – 80 % prevalence of H. pylori): most often
- duodenal ulcer(90 – 100% prevalence of H. pylori)
- gastric carcinoma: increased risk with less than 1% of pts with H. pylori developing cancer
- gastric lymphoma: H. pylori is associated with MALT
No reported association of Zollinger-Ellison syndrome with H. pylori. Zollinger-Ellison syndrome is caused by excessive gastrin production. Clinical: diarrhea, recurrent PUD, pain. Causes include gastrinoma (90%) from islet cell tumor in pancreas (90%) or duodenal wall. 50% of gastrinomas are malignant; while, 10% are associated with multiple endocrine neoplasia (MEN) I. Antral G cell hyperplasia represents 10% of causes of ZES.
(Dahnert, 4th edition: p. 690, 163; AFIP notes from Dr. Levy, GI section: p. A3 and V5)

121. What is the least likely place for intraperitoneal colon metastases?

A. lateral ascending colon
B. superior transverse colon
C. anterior rectosigmoid colon
D. superior sigmoid
E. medial cecum

Answer: A
(I think this is the best answer. The lateral ascending colon is retroperitoneum. However, see below about the paracolic gutters. Even so, I believe it is still least likely out of the choices. Dr. Raval also agreed on quick perusal )
The most common sites for pooling of malignant ascites and subsequent fixation and growth of peritoneal metastases are the pouch of Douglas (rectosigmoid), the lower small bowel mesentery near the ileocecal junction, the sigmoid mesocolon, and the right paracolic gutter. The paracolic gutters are located lateral to the attachments of the peritoneal reflections of the ascending and descending colon. The primary neoplasms that most commonly spread by this route are adenocarcinoma of the ovary, colon, stomach, and pancreas. The transverse mesocolon serves as a major route of spread from the stomach, colon, and pancreas. The gastrocolic ligament (greater omentum) is another important pathway between stomach and colon. Gastric malignancies can also extend along the gastrosplenic ligament to the spleen, whereas neoplasms in the tail of the pancreas may spread via the phrenicocolic ligament to involve the anatomic splenic flexure of the colon. Because of their continuity with the retroperitoneum, the peritoneal ligaments, in addition to serving as the avenues of intraperitoneal tumor spread, also serve as conduits of disease spread between the peritoneum and retroperitoneum
(Lee and Sagel, Computed Body Tomography with MRI Correlation, 3rd ed: Chapter 16 under Inframesocolic Compartment and Chapter 11: Colon-pathology- neoplasia- lymphoma.)

Genitourinary Radiology

122. During fetal ultrasound, a urinary tract obstruction is identified. The
most common cause of this finding in a male is:

A. obstruction at the membranous urethra
B. obstruction at the prostatic urethra
C. obstruction at the bulbous urethra
D. bladder outlet obstruction

Answer: B, they are referring to posterior urethral valves, which are in the prostatic urethra.
Posterior urethral valves are congenital thick folds of mucous membrane located in the posterior urethra (prostatic + membranous portion) distal to verumontanum. Most common cause of urinary tract obstruction and leading cause of end stage renal disease among boys (1:5000-8000).
Type I: (most common) mucosal folds (vestiges of Wolffian duct) extend anteroinferiorly from the caudal aspect of the verumontanum, often fusing anteriorly at a lower level.
Type II: (rare) mucosal folds extend anterosuperiorly from the verumontanum toward the bladder neck (nonobstructive normal variant, probably a consequence of bladder outlet obstruction)
Type III: diaphragm-like membrane located below the verumontanum (=abnormal canalization of urogenital membrane)
(Dahnert, 4th ed: p. 784)


123. A patient with known bladder cancer undergoes a contrast enhanced CT of the abdomen and pelvis. There is a 2 cm adrenal mass, which measures 30 HU. What should be done next?

A. nothing, tell the clinician the patient has metastatic disease
B. get delayed images to confirm the HU
C. nuclear medicine study with NP-59-cholesterol

Answer: B.
Unilateral Adrenal Mass
CT attenuation <0 HU = benign mass
0-15 HU = probably benign
>15 HU = indeterminate
on 15- minute delayed CECT scan: < 25 HU benign lesion
> 25 HU malignant lesion
Cause: rapid contrast washout from benign lesions
(Dahnert, 4th ed: p.740)

124. The most common risk factor/predisposing factor for the development of contrast induced renal failure is:

A. pheochromocytoma
B. myeloma
C. pre-existing renal insufficiency

Answer: C
Acute Renal Failure may occur suddenly with rapid deterioration of renal function. An increase in serum creatinine of >25% or to > 2mg/dl within 2 days of receiving contrast material. The frequency is 1-30 % and represents the 3rd most common cause of in-hospital renal failure after hypotension and surgery. Risk factors:
1. Preexisting renal insufficiency (serum creatinine > 1.5 mg/dL)
2. Diabetes mellitus (possibly related to dehydration/hyperuricemia)
3. Dehydration
4. Cardiovascular disease
5. use of diuretics
6. advanced age > 70 years
7. multiple myeloma (in dehydrated patients)
8. hypertension
9. hyperuricemia /uricosuria
(Dahnert, 4 th ed.: p.940)

125. CT with and without contrast demonstrates a 3 cm mass in the midpole of a kidney with a bifid system. The attenuation of the mass is equal to that of renal parenchyma. The most likely etiology is:

A. Oncocytoma
B. Renal cell carcinoma
C. Abscess
D. Normal (column of Bretin)

Answer: D
Large column of Bertin represents a large septum/cloison of Bertin or focal cortical hyperplasia or benign cortical rest or focal renal hypertrophy. It is persistence of normal septal cortex/excessive infolding of cortex usually in the presence of partial or complete duplication (bifid system). Its location is between the upper and interpolar portion. It is usually a mass <3 cm in largest diameter. The mass is continuous with renal cortex with enhancement pattern and echogenicity similar to renal cortex.
Oncocytomas have no association with bifid systems. In addition, its average size is 6-7.5 cm, and the tumor has homogeneously low attenuation/ hypoechogenicity (50%).
(Dahnert, 4th ed: p.730 and 780)

126. During an IVP an increasingly dense nephrogram is seen. All of the
following are causes except:

A. Systemic hypotension
B. Chronic glomerulonephritis
C. Contrast nephropathy
D. Acute ureteral obstruction

Answer: B
Chronic glomerulonephritis results from acute post streptococcal glomerulonephritis. It presents with small smooth kidneys with wasted parenchyma with normal papillae and calices. It shows patchy nephrograms with diminished density of contrast.
Causes of increasingly dense nephrogram:
A. Vascular = diminished perfusion
1. systemic arterial hypotension (bilateral)
2. severe main renal artery stenosis (unilateral)
3. acute tubular necrosis (33%): due to contrast material nephrotoxicity
4. acute renal vein thrombosis
B. Intrarenal
1. acute glomerular disease
C. Collecting system
1. intratubular obstruction
a. uric acid crystals (acute urate nephropathy)
b. precipitation of Bence-Jones protein (myeloma nephropathy)
c. Tamm-Horsfall protein (severely dehydrated infants/children)
2. Acute intrarenal obstruction: ureteral calculus
(Dahnert, 4th ed: p.763, 733)

127. An 8-year-old girl presents with fever, urinary frequency with WBC's in her urine. What is the next step in her management?

A. IVU
B. Contrast enhanced CT of the abdomen and pelvis
C. Renal/bladder ultrasound
D. Voiding cystourethrogram and nuclear medicine cystogram

Answer: C
< 5 y/o VCUG
DMSA
>5 y/o U/S
VCUG
U/S looks for structural abnormalities, and should be obtained in all children after the first UTI.
VCUG should be obtained in ALL children with UTI <4 yrs old to look for vesicoureteral reflux. It should be carried out or all older children with abnormal U/S, bladder dysfunction, or repeated UTI.
Renal cortical scintigraphy has the highest sensitivity and specificity of imaging modalities and should be obtained if results would affect management.
(Dahnert, 4th ed: p. 725 and Primer, 2nd ed: p.777)

128. The most common cause of ureteropelvic obstruction is:

A. intrinsic ureteral abnormality
B. vascular compression
C. external fibrous band

Answer: A
Congenital ureteropelvic junction obstruction is the most common congenital anomaly of the GU tract in neonates. 20% of obstructions are bilateral. Causes fetal/neonatal hydronephrosis. Treatment is pyeloplasty.
Causes: Intrinsic, 80%: defect in circular muscle bundle of renal pelvis
Extrinsic, 20%: renal vessels (lower pole artery or vein)
Radiographic features: -caliectasis, pelviectasis, no dilation of ureter, delayed contrast excretion of kidneys, and poor opacification of renal parenchyma.
M:F = 5:1; Location: left > right, bilateral (10-40%)
(Primer p. 773; also see Dahnert, 4th ed: p. 816 for more extensive list of causes)

129. All of the following are seen with aldosteronism except:
A. hyperkalemia
B. hypokalemia
C. low rennin

answer: A
Adrenal adenomas may secrete excessive hormone and cause one of the endocrine syndromes or be non-hyperfunctioning and present as an unsuspected adrenal mass. Non hyperfunctional adenomas are incidental findings in as much as 3-5% of the population. Function of an adenoma cannot be determined by imaging appearance but is assessed clinically. With aldosterone-secreting adenomas, patients typically present with diastolic hypertension without edema, hyposecretion of renin that fails to increase appropriately during volume depletion, and hypersecretion of aldosterone that fails to suppress appropriately during volume expansion. This hypersecretion of aldosterone leads to the continual renal tubular exchange of sodium and potassium resulting in the hypernatremia and hypokalemia which is characteristic of this disease.
(References: 1998 old test question #91; Brandt and Helms, 2nd ed: p. 770; Harrison's Principles of Int Med: p. 1366.)

130. Helical CT which is positive for a ureteral calculus demonstrates which of the following positive findings:

A. perinephric hemorrhage
B. hyperattenuating lesion with a hypoattenuating center

Answer: none of the above
Hyperattenuating lesions with a hypoattenuating center sounds more like a phlebolith than calculus.
Perinephric hemorrhage is not commonly associated with ureteral calculus. Perinephric stranding is a common secondary sign on CT. It appears as filiform, curvilinear areas of soft-tissue attenuation in the perinephric space. These are attributed to venous collateral vessels in the perinephric space, bridging septa in the compartment which serve as conduit for the spread of fluid, inflammation, or neoplasm from the kidney, and a network of lymphatic channels. Perinephric stranding seen at CT is probably a manifestation of the increased pressure in the collecting system during the acute phase of ureteral obstruction, which in turn causes increased lymphatic flow secondary to pyelolymphatic backflow. Pyelosinus and pyelovenous backflow of urine may also contribute.
Secondary CT findings such as stranding of perinpehrc fat, hydronephrosis, the tissue rim sign, and nephromegaly in the diagnosis of ureterolithiasis has been documented.
(Takahashi, N. et al. Ureterolithiasis: Can clinical outcome be predicted with unenhanced Helical CT. Radiology 1998;208:97-102)


131. CT scan of the abdomen and pelvis is performed in a 50-year-old male. There is a 2 cm renal mass that enhances to 60 HU. The next step in
management is:

A. nothing more necessary
B. fine needle aspiration
C. nephrectomy
D. HMPAO nuc study

Answer: C
This question is not clear what the pre-contrast CT Hounsfield units of the renal mass was. No other foci are identified in the CT of the abdomen and pelvis to suggest this could be metastasis. In addition, the other kidney is clear of lesions. In this case of a 50 yr old male, nephrectomy (partial or total) would be best. …Accurate diagnoses of renal masses can be made with pre- and postcontrast CT images. Percutaneous renal biopsy is rarely indicated, not because it is inaccurate, but because in most instances it is superfluous. Their CT or MRI appearance definitively diagnoses most cases of RCC, and they require surgical removal. The few very small or complex lesions that are indeterminate are those that are particularly difficult to diagnose by biopsy or angiography as well. Biopsy, however, may be useful in cases of unresectable, apparently malignant tumor of uncertain histology (e.g., metastasis or lymphoma versus RCC), where treatment may be altered by knowledge of the cell type. In such a population, a sensitivity of 80% and a specificity of 100% with minimal complications have been reported. In that study, all of the patients with RCC with metastases at the time of presentation died within 1 year
Nephron-sparing surgery is increasing in frequency, because excellent results have been reported with small, low stage RCC
(Lee and Sagel, Computed Body Tomography with MRI Correlation, 3rd ed: Chapter 18. The Kidney. Malignant Renal Neoplasms under Angiography and Biopsy)

132. A mass is identified within the renal pelvis in a patient with known analgesic abuse. Which of the following is the most likely diagnosis?

A. squamous cell carcinoma
B. transitional cell carcinoma
C. fungus ball

Answer: B
Analgesic nephropathy is related to excessive ingestion of phenacetin, acetaminophen, aspirin or NSAIDS in combination or alone, resulting in interstitial nephritis and papillary necrosis. Direct toxicity to the collecting tubules, generalized mircroangiopathy in the peritubular vessels, endothelial necrosis in the interstitial cells and platelet aggregation in the vasa recta occur over time and lead to necrosis of the renal papillae. Sloughed papillae may cause ureteral obstruction. Transitional cell dysplasia predisposing to neoplasia is widespread. Transitional cell carcinoma demonstrates an increased prevalence in this patient population, especially in the upper tracts, than in the general public. More so than squamous cell carcinoma. If sloughed papillae were one of the choices, it would be the correct answer as this occurs much more commonly than TCC in this disease entity.
(Dunnick, Textbook of Uroradiology, p. 282; Dahnert's, 4th ed: p. 761; see question # 15 in old 1998 test- answers by Ronit.)

133. A 25-year-old male has a one-year history of nephritic syndrome. He develops sudden flank pain and macroscopic hematuria. On CT the involved kidney is enlarged with faint opacification. The most likely diagnosis is:

A. urinary tract obstruction
B. acute gouty nephropathy
C. renal vein thrombosis
D. xanthogranulomatous pyelonephritis (XGP)


Answer: C ?
I wonder if the question was nephritic syndrome or nephrotic syndrome. In simple terms, nephritic conditions are associated with increased numbers of RBC in the urine, whereas nephrotic conditions are associated with increased urinary protein excretion. Nephritic syndrome can become nephrotic in nature. In adults, the most common underlying abnormality of renal vein thrombosis is membranous glomerulonephritis (which I believe is seen in nephrotic syndrome). Approximately 50% of patients with membranous glomerulonephritis have renal vein thrombosis. ?Urinary tract obstruction? I am not sure, but this could be true. But more is written about renal vein thrombosis associated with glomerulernephritis.
Acute gouty (or urate) nephropathy is related to a complication of chemotherapy or radiation therapy causing increased nucleoprotein catabolism and oliguric acute renal failure. An increasingly dense nephrogram with enlarged kidneys and absent or markedly diminished pyelogram is seen on urography.
XGP usually have symptoms of long duration. If fulminant , then very sick.
(Dunnick, 3rd ed: p. 201-202, 233)


134. Cowper's glands:
A. are located in the urogenital diaphragm/membranous urethra
B. empty into the prostatic urethra

answer: A
Cowper’s glands are two pea-sized bulbourethral glands lying laterally and posteriorly between fasciae and sphincter urethra within urogenital diaphragm. They empty into the mid bulbous urethra inferiorly on both sides of midline.
(Dahnert, 4th ed: p.756)

135. Regarding the use of metformin hydrochloride and its interaction with radiographic contrast material, which of the following is true?

A. contrast material should not be given if the medication has been taken
within the last 48 hours
B. it is directly nephrotoxic
C. the combination of the two results in a lactic acidosis that is nephrotoxic

answer: C (I believe the concern is of metformin-associated lactic acidosis in the susceptible patient (0-0.084 cases per 1000 patient-yrs) with mortality in reported cases about 50%. I am not sure it is necessarily nephrotoxic.)
Metformin (Glucophage) is excreted unchanged by the kidneys, probably by both glomerular filtration and tubular excretion. The renal route eliminates approx. 90% of the absorbed drug within the first 24 hrs. It appears to cause increased lactic acid production by the intestine. Any factors which decrease metformin excretion or increase blood lactate levels are, therefore, important considerations in terms of lactic acidosis (renal insufficiency being a major consideration). Radiographic contrast agents are not an independent risk factor for patients taking metformin, but rather are a concern only in the presence of underlying renal failure. Of metformin-associated lactic acidosis cases reported worldwide (yrs 1968-1991), 7 of the 110 patients received iodinated contrast material prior to developing lactic acidosis. Therefore, Metformin (Glucophage) should be discontinued for 48 hrs after contrast medium administration (accumulation of metformin may result in lactic acidosis which is fatal in 50%), and reinstated only after renal function has been re-evaluated and found to be normal.
(Dahnert, 4th ed: p.940; handout of ACR Manual on Iodinated Contrast Media, 3rd ed. by ACR committee: p. 25-26)

136. Question regarding non-occlusive mesenteric ischemia not well remembered...

A. angiogram shows narrowing at the origin
Nonocclusive mesenteric ischemia (NOMI) has been reported increasingly after cardiac surgery and dialysis along with the other causes of acute mesenteric ischemia. It refers to a state of hypoperfusion that exists in the absence of thrombotic or embolic vascular occlusion. It is synonymous with splanchnic vasoconstriction. The most common causes are cardiac failure, sepsis, and other severe systemic illnesses that result in shock. Little controversy exists over angiography as the only way to diagnose NOMI before intestinal infarction occurs. The characteristic angiographic pattern is narrowing of the origins of major branches of the SMA. Additionally, there may be tapering, beading, or narrowing of intestinal branches and spasm of intestinal arcades with reduced or absent filling of intramural vessels. Moreover, there is general agreement that when NOMI has been identified, infusion of a vasodilator, most commonly papaverine hydrochloride, into the SMA can relieve the vasoconstriction and prevent further ischemic damage to the bowel. Studies showed that this approach reduced mortality rates of NOMI from historical levels of 70% to greater than 90% to 0-55%. Although, these series were small (a total of 33 patients), there are a number of other isolated reports of survival with and without the need for bowel resection after papaverine infusion. Pts with peritoneal signs and the angiographic diagnosis of NOMI undergo laparotomy and receive intra-arterial infusion of papavarine before, during, and after surgery as indicated.
(Perler and Becker, Vascular Intervention: A Clinical Approach, 1998: p. 586-529; Brandt, LJ et al, AGA Technical Review on Intestinal Ischemia. Gastroenterology 2000;118: p. 954-968.)

137. A patient is S/P MVA with superior and inferior pubic rami fractures, an ipsilateral SI joint fracture. He has a mass in the pelvis. There is no blood at the urethral meatus. The next study should be:

A. IVP
B. Contrast enhanced CT of the abdomen and pelvis
C. Retrograde urethrogram

Answer: C
Need to determine the patency of the urethra prior to any manipulation if a male. Urethral trauma is seen in 4-17% of pelvic fractures in males and in <1% in females. There is associated bladder injury in 20%. The single most important clinical sign of urethral injury is blood at the external meatus. Even without blood at the meatus, certain types of urethral injuries are possible (Type I for instance). If the RUG is negative, then a cystogram should be performed.
(Dahnert, p. 817)


MRod Answers

156.
A 4-year-old boy presents with fever, malaise and cervical adenopathy. CXR demonstrates cardiomegaly. The most likely cause is:
A. Viral Syndrome.
B. Mycoplasma.
C. Kawasaki’s Disease.
D. Other
The correct answer is C.

Most common organisms for viral pneumonia are Rhinovirus (43%), RSV (12%), and Mycoplasma (10%). Viruses are the most common cause of pneumonia in children under 5yo. Chest radiograph demonstrates air trapping with interstitial opacities, peribronchial cuffing. Viral pneumonia or pharyngitis should not cause cardiomegaly.
In patients with mycoplasma infection, 50% get tracheobronchitis, 30% get pneumonia, 10% get pharyngitis, and 10% get otitis media. Chest radiographs demonstrate segmental, subsegmental, or reticulonodular interstitial infiltrates. Lobar involvement may be seen in the lower lobes. 20% get pleural effusions. Treat with supportive care with slow improvement.
Kawasaki’s disease is an acute febrile multisystem vasculitis of unknown cause with a predilection for the coronary arteries. Associated with polyarthritis (30-50%), aseptic meningitis (25%), hepatitis, pneumonitis, mucosal reddening, cervical lymphadenopathy (82%), maculopapular rash on extensor surfaces (99%), bilateral nonpurulent conjunctivitis (96%), and erythema of palms and soles with desquamation (88%). Coronary artery manifestations include aneurysms, stenosis, and occlusions. Cardiac manifestations include myocarditis, pericarditis, valvulitis, and A-V conduction disturbances. Patients can develop intestinal pseudoobstruction and transient gallbladder hydrops. 0.4-3% mortality from coronary/cardiac complications. Treated with aspirin and IgG.


157.
Which of the following is NOT seen in Down’s syndrome during fetal ultrasound?
A. Nuchal Fold Thickening.
B. Duodenal Atresia.
C. Echogenic Bowel.
D. Mild Cerebral Ventriculomegaly.
E. Mild Pelvicaliectasis.
The correct answer is E.

Occipital-nuchal skin thickening >6mm on transcerebellar diameter view is seen in 40-70%. The double bubble sign of duodenal atresia is not apparent before 22 weeks GA. Hyperechogenic is seen at <20 weeks GA.

Other findings on OB-US with Down’s Syndrome:
Cystic Hygroma
Ratio of measured to expected femur length 0.91.
Ratio of measured to expected humerus length 0.90.
Elevated BPD/femur ratio (due to short femur)
Hypoplasia of middle phalanx of 5th digit resulting in inward curve.
Separation of great toe.
VSD/Complete AV canal.
Omphalocele.
IUGR.


158.
What is the most common cause of uterine bleeding in a post-menopausal woman?
A. Endometrial polyp.
B. Hyperplasia of the endometrium.
C. Endometrial carcinoma.
D. Endometrial atrophy.
The correct answer is D.

In postmenopausal women, an endometrium <5mm thickness is normal in symptomatic patients and an endometrium <9mm thickness is normal in asymptomatic patients. A thin endometrium with cystic changes is consistent with the diagnosis of cystic atrophy. Patients on unopposed estrogen are at increased risk for endometrial polyps, hyperplasia, or carcinoma. Any of the answers can cause postmenopausal bleeding. Patients with endometrial polyps are usually asymptomatic.


159.
Which of the following is FALSE regarding the carotid artery Doppler examination?
A. In a patient with hypertension and marked carotid stenosis, velocities may be elevated in the contralateral carotid.
B. Bradycardia may result in elevated systolic and diastolic velocities.
C. The distal stenosis in a tandem lesion may have decreased velocity.
I think the correct answer is A.

According to Dahnert, higher velocity measurements can be caused by HTN, severe bradycardia, and obstructive contralateral carotid disease. If question A really said marked and not occlusive carotid artery disease, then the answer would be FALSE. There was no mention of contralateral increase in velocities for severe stenosis up to 99%. Lower velocity measurements can be caused by arrhythmia, aortic valvular disease, severe cardiomyopathy, proximal obstructive lesion >95% (tandem lesion).


160.
Which of the following is FALSE regarding asymmetric intrauterine growth retardation?
A. The head:body ratio becomes increased. TRUE.
The Head Circumference/Abdominal Circumference and Femur Length/Abdominal Circumference ratios become increased because the head size and femur length are less affected than the abdomen.
B. Associated with oligohydramnios. TRUE.
Oligohydramnios is defined as an amniotic fluid index <5cm.
C. Fetal carotid diastolic flow is increased. FALSE.
I found no mention of this so I assume it is false. Asymmetric IUGR can result in and elevated umbilical artery systolic/diastolic ratio.
D. Amniotic fluid index >20 may be an associated finding. FALSE.
Normal amniotic fluid index is between 5 to 20cm.

Asymmetric IUGR is a disproportionate reduction of fetal measurements due to uteroplacental insufficiency with preferential shunting of blood to the fetal brain occurring after 26 weeks GA.

Also associated with:
Abdominal circumference >2 SD below the mean for age is highly suspicious. AC >3 SD below the mean for age is diagnostic.
Accelerated placental maturity.
Low estimated fetal weight.
Maternal Hypertension.
Small BPD.
Slow BPD growth rate.


161.
Regarding adenomyosis, which of the following is true?
A. See high signal intensity on T1 weighted images. FALSE?
I am not sure what the wording for the question really was. MRI is the modality of choice to diagnose adenomyosis. The myometrial mass of adenomyosis is primarily low signal on all sequences. Occasionally there is high signal intensity spots on T1 and T2 corresponding to hemorrhagic areas.
B. Endometrium is present in the adnexa. TRUE?
Once again, it depends on the exact wording of the question. Adenomyosis is a focal or diffuse invasion of the myometrium by endometrium which incites myometrial hyperplasia. Adenomyosis is associated with endometriosis in 36 to 40% of cases.
C. Treated by local resection. TRUE.
The treatment is hysterectomy.


162.
Which finding on US of the thyroid is most indicative of malignancy?
A. Egg shell calcifications.
B. Speckled calcifications.
C. Heterogeneity.
D. Cystic regions.
I think the correct answer is B.

Microcalcifications measuring <1mm favor a malignancy and are seen in 54% of thyroid neoplasms.
Benign appearing calcifications include coarse and peripheral eggshell calcifications.

Other indicators that favor a malignant thyroid nodule include:
Irregular contour
Poor margination
Size > 4cm
Single cold nodule on radionuclide scan
Age < 20yo
History of neck irradiation, especially in childhood
Family history of thyroid malignancy, especially medullary carcinoma
Male


163.
The size of an ovarian follicle just prior to rupture is:
A. 5mm
B. 10mm
C. 15mm
D. 20mm
I think the correct answer is D.

According to Dahnert, the physiologic Graafian follicle can reach up to 25cm at midcycle.


164.
The middle hepatic vein:
A. Divides the right anterior and posterior lobes.
B. Divides the left medial and lateral lobes.
C. Divides the right and left lobes.
The correct answer is C.

The right and left lobes are separated by the major lobar fissure, defined by the middle hepatic vein. The right hepatic lobe is subdivided into the anterior and posterior segments by the right intersegmental fissure, defined by the right hepatic vein. The left hepatic lobe is subdivided into medial and lateral segments by the left intersegmental fissure, defined by the left hepatic vein and the fatty fissure of the ligamentum teres and falciform ligament.
I would recommend reviewing the Couinaud system.



165.
During pelvic ultrasound of a post-menopausal woman, a 3.5cm ovarian cyst is seen. What should be done next?
A. Follow up ultrasound in 3 months.
B. Nothing.
C. Schedule the patient for laparoscopic surgery.
D. Biopsy.
The correct answer is A.

Postmenopausal:
<5.0cm and simple – Follow up in 3 months, then 6 months x 2, then 1 year.
>5.0cm or complex at any size – Laparoscopy and resection.

Premenopausal:
<2.5cm and simple – Physiologic follicle. No follow up.
>2.5cm to 10cm (simple or complex) – Follow up in 6 weeks x 2.
>10cm – Laparoscopy and resection.


166.
Which of the following has an echogenic renal cortex?
Alport’s syndrome. TRUE
Neonatal kidney. TRUE
Acute glomerulonephritis. TRUE

Alport’s syndrome consists of chronic hereditary renal disease, deafness, and ocular abnormalities. Some patients may develop cortical nephrocalcinosis with an echogenic cortex that does not shadow.
The neonatal kidney is equal in echogenicity to the liver.
Acute glomerulonephritis results in a hyperechoic renal cortex.


167.
Which of the following is true regarding ultrasound of the uterine endometrial stripe?
A. Endometrial stripe is best imaged in the coronal plane.
B. Measurement should include the surrounding hypoechoic region.
C. Fluid indicates malignancy in a post-menopausal patient.
D. Tamoxifen causes increased endometrial thickness with cystic changes.
The correct answer is D

The endometrial stripe is best images in the midline sagittal plane. The hypoechoic region surrounding the endometrium is the myometrium and should NOT be included in the endometrial measurement. A small amount of fluid in the endometrial cavity is normal in asymptomatic patients. Larger amounts of fluid may be associated with benign conditions, most often related to cervical stenosis, or with malignancy.

Tamoxifen, a non-steroidal antiestrogen, is used for adjuvant therapy in pre- and post-menopausal women with breast cancer. Tamoxifen competes with the estrogen receptor. It has anti-estrogen effects in premenopausal women and weak estrogenic effects on postmenopausal women. Tamoxifen therapy causes cystic changes with thickening of the endometrium and increases the womans risk for endometrial cancer, hyperplasia, and polyps.


168.
Regarding the appearance of a rotator cuff tear on ultrasound, which of the following is FALSE? (not well remembered)
A. Fluid is identified in the subacromial and subdeltoid bursa. FALSE
B. Rotator cuff is not seen. TRUE
C. Rotator cuff only partially seen. TRUE
D. Focal thickening seen. FALSE
E. Hypoechoic and hyperechoic areas are identified. TRUE

Ultrasound criteria for rotator cuff tear:
*Non-visualization of the cuff. Seen in massive tear.
*Focal non-visualization of the cuff. Seen in small tears.
*Discontinuity in the cuff. Seen in proximal tears that are more proximal in the tendon. These small defects fill with fluid or hypoechoic reactive tissue.
*Focal abnormal echogenicity. Diffuse abnormal echogenicity is an unreliable sign. It can be hyperechoic or hypoechoic.

Associated ultrasound findings in rotator cuff tear:
*Subdeltoid bursal effusion. The most reliable associated finding. NOT subacromial effusion.
*Joint effusion.
*Concave subdeltoid fat contour.
*Bone surface irregularity.


169.
On a fetal sonogram, the aorta and pulmonary arteries have a parallel course, the fetus most likely has:
A. Transposition of the great vessels.
B. Tetralogy of Fallot.
C. Truncus arteriosus.
D. Hypoplastic left heart.
The correct answer is A.

TGV is best seen in the short axis view.
In TOF, the VSD, dilated aorta, pulmonary atresia/stenosis, and dilated pulmonary artery can be seen. Right ventricular hypertrophy rarely occurs in utero.
In TA, a large truncal artery is seen overriding the VSD.
In HLH, a small ventricle is seen. The mitral valve is typically hypoplastic or atretic, as is the aorta.
Color Doppler is very helpful to diagnose all of these.


170.
Which of the following is FALSE regarding parovarian cysts?
A. Can undergo torsion.
B. Often located in the broad ligament.
C. Mullerian duct remnant.
D. Are not under hormonal influence.
E. Account for 10% of adnexal cysts.
The correct answer is C.

Parovarian cysts can undergo torsion and even rupture. The are located in the broad ligament. They are of mesothelial or paramesonephric origin. Rarely, they are of mesonephric origin. They are NOT under hormonal influence and do NOT undergo cyclical change. They also account for 10% of all adnexal masses.


171.
Regarding peripheral AV fistulas
A. Many will spontaneously close/resolve. TRUE
B. There is increased velocity of flow distal to the fistula. TRUE
C. There is loss of the normal triphasic waveform in the artery proximally. TRUE

These things are tough for me to keep straight so maybe this helps you:

AVM’s are congenital lesions that occur during the retiform stage of vascular development. They slowly enlarge over the patient’s lifetime. Most AVM’s have a primary component: arterial, venous, lymphatic, or capillary. The flow pattern depends on the type. Many are associated with syndromes.

Hemangiomas are vascular malformations that occur during the capillary stage of vascular development. These benign vascular tumors are present in infancy, proliferate over time, and usually involute during adulthood. Located in mucous membranes, skin, and internal organs. Capillary hemangiomas are in a growth phase. Cavernous hemangiomas are in a quiescent stage and characterized by large vascular channels. In solid organs, hemangiomas have normal caliber feeding vessels and early filling of vascular spaces that persists through the venous phase.

Telangiectasias are focal lesions composed of dilated arterioles, capillaries, and venules. They are typically located on the skin and mucous membranes but can also be seen in the visceral organs. (i.e. Hereditary hemorrhagic telangiectasia).

AV Fistulas are almost always acquired connections between and artery and vein from accidental or iatrogenic trauma. There is a 0.01% risk from femoral artery catheterization. AV fistulas may close spontaneously or enlarge over time and many will. Patients may present with local symptoms, distal ischemia from steal, or high output failure. Rarely, fistulas are congenital.

AV shunts are normally present in many vascular beds. These physiologic shunts become quite prominent and can be seen n certain disease states such as cirrhosis, and hypervascular tumors.


172.
During fetal ultrasound, enlarged cystic kidneys are identified with oligohydramnios. Which is the most likely diagnosis?
A. Multicystic dysplasia.
B. Autosomal recessive polycystic kidney disease.
C. Autosomal dominant polycystic kidney disease.
D. Medullary cystic disease.
The correct answer is A.

Multicystic Dysplastic Kidneys (MCDK) result from early (8-10 weeks) severe urinary obstruction causing maldevelopment of the renal parenchyma which is replaced by cysts of variable size. The kidneys are enlarged. If the contralateral kidney is normal, the bladder is seen and the AFI is normal, there is a good prognosis. In 40% of cases, there is associated contralateral renal abnormalities (i.e. bilateral MCDK(20%), renal agenesis (10%), UPJ obstruction (7%), and renal hypoplasia). Fetuses with MCDK, severe oligohydramnios, non-visualization of the bladder have a poor prognosis with likely lethal renal disease.
ARPKD demonstrates large kidneys with very small cysts that are usually too small to be seen by US creating increased echogenicity. May have oligohydramnios depending on renal function.
ADPKD occasionally presents in utero and has a variable appearance on US. Renal enlargement is the most common finding. The kidneys may or may not have cysts and are have increased echogenicity. AFI is usually normal. Recurrence risk in family is 50%.
Medullary Cystic Disease has a childhood and adult form. The kidneys are small to normal sized as a result of progressive renal tubular atrophy and fibrosis. Typically demonstrates echogenic kidneys with cysts in the medulla and corticomedullary junction.